You are on page 1of 107

MODEL QUESTION PAPER

FINAL EXAMINATION
SESSION : 2023 - 24
SUBJECT : ENGLISH CORE
CLASS - XI
Time Allotted : 3 Hours Maximum Marks : 80
SECTION – A (READING)
1. Read the passage carefully. (10)
Red Cross: How Hope Evolved
On June 24, 1859, Emperors Napoleon III and Franz Joseph I engaged in the Battle of
Solferino, commanding a combined total of about 270,000 troops onto the field for a
single day of battle. Nearly 40,000 were either dead, injured, or missing, many of whom
were simply left to die on the battlefield. Later, spectators crowded the fields, looking for
loved ones, searching for items they could sell, or simply taking in the horrors of the
battle. A Swiss businessman and social activist Jean Henri Dunant, who was traveling in
Solferino witnessed all this.
Jean Henri Dunant witnessed the atrocities of war as well as the countries not prepared
or equipped to ease the suffering of those who had been injured in the Battle of Solferino.
Dunant organized a group of volunteers to help bring water and food to the injured, to
assist with medical treatment, or write letters to the families of those who were dying and
he urged the public to create an organization which would assist the wounded, regardless
of which side they fought for during times of war. After that moment, he wrote the book,
A Memory of Solferino, which urged the public to create an organization which would
assist the wounded, regardless of which side they fought for during times of war. His
writing inspired countless others to rally behind him in the creation of the International
Federation of the Red Cross.
The modern-day Red Cross began by devoting itself largely to disaster relief and
epidemic treatment. This effort continues to this day. One of the easiest ways you can
help the Red Cross is to make sure you are able to donate blood and make an appointment
at the Red Cross website. But, donating blood isn't the only way you can help out—the
Red Cross also encourages donating your time if you can. This is what the Red Cross
wants everyone to know.
On the basis of your reading of the passage given above, answer the following
questions. (1 × 10 = 10)
a) How many soldiers fought in the battle of Solferino on June 24?
i) 270,000
ii) 40,000
iii) 230,000
iv) 23,000

1
b) What is the correct order of the information given below:
I) Dunant organised a camp of volunteers.
II) Emperor Napoleon III and Franz Joseph I fought a battle.
III) The book ‘A Memory of Solferino was written.
IV) Dunant travelled in Solferino.
i) IV, III, II, I
ii) I, II, III, IV
iii) II, I, IV, III
iv) II, IV, I, III
c) The writer of 'A Memory of Solferino’ was:
i) Emperor Napoleon III
ii) Franz Joseph I
iii) Jean Henri Dunant
iv) International Red Cross
d) The modern-day Red Cross does not deal with:
i) Blood donation
ii) Disaster Relief
ii) Epidemic Treatment
iv) Writing a book
e) The most appropriate sub-heading to para 2 of the passage is:
i) How International Federation of Red Cross Evolved!
ii) Functions of Red Cross
iii) The Contributions of Jean Henri Dunant
iv) The Battle of Solferino
f) The aid provided by Dunant’s volunteers during the battle of Solferino was:
I) to help bring water and food to the injured
II) to assist with medical treatment
III) to write a book
IV) to give relief in epidemic
i) I and III
ii) III and IV
iii) I and II
iv) II and III
g) You can donate your blood and your _________ to Red Cross.
h) Jean Henri Dunant participated in the war. (True/ False)
i) Find an antonym for the word 'Atrocities' as used in the passage.
i) Honors
ii) Cruelties
iii) Barbarities
iv) Kindness

2
j) Choose a synonym for the word 'Devoting' as used in the passage.
i) Neglecting
ii) Dedicate.
iii) Withholding
iv) Disregarding
2. Read the following passage: (8)
India Covid-19 numbers explained
1. With novel Coronavirus spreading rapidly all over the country, there are only three
states right now, Meghalaya, Sikkim and Andaman and Nicobar Islands, that have
less than 1,000 people infected with the disease.
2. Lakshadweep, of course, still hasn’t reported even a single case till now, the only
region in India entirely free of the epidemic.
3. Otherwise, even the relatively smaller states now have significantly large spread of
the disease. Goa, for example, has seen more than 7,000 of its people infected by
the virus till now. Tripura has over 5,500 cases, while Manipur has more than
3,000, and Nagaland a little less than 2,500. Puducherry has more than 4,000 cases,
while even Daman and Diu has over 1,300 people infected.
4. And in each of these states, the numbers are rising at a fast pace, at a rate higher
than the national level. The infections had initially reached these states in the first
and second week of May, when the lockdown was relaxed for the first time to
enable people stuck in different parts of the country to return to their native places.
5. After a period of very slow growth, the number of cases have begun to rise rapidly
in the last one month. In Goa, for example, the total number of infected people has
nearly doubled in the last 15 days. Same has happened in Puducherry, as well.
6. Tuesday was one of those rare occasions when the number of active cases in the
country, those who are yet to recover from the disease, went down compared to the
previous day. That is because the number of recoveries, combined with the number
of deaths, exceeded the new cases that were detected on Tuesday.

7. With over 52,500 new cases detected in the country, the total number of infections
crossed 19 lakh, out of `which 12.82 lakh people have recovered from the disease.
The number of dead is now close to 40,000.

3
8. The number of recoveries on Tuesday was the highest-ever for a single day. More
than 51,700 people were declared to have been recovered. Three days earlier, the
number of recoveries had crossed 50,000 for the first time, but in the next two days
the number had fallen to much lower levels.
Answer the following questions: (1 × 8 = 8)
a) How are these three states – Meghalaya, Sikkim and Andaman and Nicobar Islands
– different from the rest of India?
i) they are the only states to have less than 1,000 people infected with novel
coronavirus
ii) they are three of the five states to have less than 1,000 people infected with
novel coronavirus
iii) they are the only states to have less than 2,000 people infected with novel
coronavirus
iv) none of these
b) __________ is the only region in India which is entirely free of the epidemic.
i) Andaman and Nicobar Islands
ii) Maharashtra
iii) Lakshadweep
iv) Tripura
c) What is common among Goa, Tripura, Manipur, Nagaland, Puducherry and Daman
and Diu?
i) the numbers are rising at a fast pace at rates lower than the national level
ii) the numbers are falling at a fast pace at rates equal than the national level
iii) the numbers are rising at a fast pace at rates higher than the national level
iv) none of these
d) What is the meaning of the word ‘novel’ in the first paragraph?
e) Which date in the graph shows the highest jump of detected cases in a day?
i) July 30
ii) July 31
iii) August 1
iv) August 2
f) Of the 19 lakh infected cases, how many have recovered?
i) 11.82 lakh
ii) 13.82 lakh
iii) 12.81 lakh
iv) 12.82 lakh
g) When had the number of recoveries crossed 50,000 for the first time?
i) two days earlier
ii) Tuesday
iii) three days earlier than Tuesday
iv) none of these
h) Which word in the passage is opposite in meaning to “indigenous”?
i) rapidly
ii) significantly
iii) native
iv) rare

4
3. Read the following passage carefully.
It’s 10 pm and the research paper is due the next morning. Sam types frantically. Two
weeks ago, it seemed that there was plenty of time to get the paper done. Last week, the
final of a soccer match on TV made it hard to study. Now it’s crunch time. Looking at
the clock, Sam wonders, “Why do I keep doing this to myself ? Why haven’t I learned
not to put things off until the last minute ?”
The word procrastination comes from the Latin term ‘Procrastinatus’. It means to put
forward until tomorrow. Standard dictionary definitions all include the idea of
postponement or delay. Steel, a psychologist who has reviewed hundreds of studies on
the subject, states that to procrastinate is “to voluntarily delay an intended course of
action despite expecting to be worse-off for the delay”. Another expert, Dr. Joseph R.
Ferrari (2005), distinguishes between people who tend to put things off and “chronic” or
“real” procrastinators for whom this is their life and who might even need therapy.
Ferrari categorizes procrastinators into three types: (a) stimulation types that get a thrill
from beating a deadline, (b) avoiders put off doing things that might make others think
badly of them, and (c) decisional procrastinators postpone making a decision until they
have enough information to avoid making a wrong choice. Chronic procrastinators tend
to have a low self-esteem and focus on the past more than the future. The Discounted
Expectancy Theory illustrates with a student like Sam who puts off writing a paper. When
the deadline is far off, the rewards for socializing now are greater than those for finishing
a task not due until later.
As the deadline looms, the rewards, or consequences for finishing the paper become more
important. Tice and Baumeister (1997) found that procrastinators on the average got
lower grades and had higher levels of stress and illness. Chu and Choi (2005) however,
say that not all procrastinators are lazy and undisciplined. “Passive procrastinators” are
more stressed, less efficient. “Active procrastinators prefer to work under pressure” and
“if something unexpectedly comes up, they will knowingly switch gears and engage in
new tasks they perceive as more urgent.”
a) Based on your reading, make notes using appropriate abbreviations and supply a
suitable title. (5)
b) Write a summary of the above passage in 50 words. (3)
SECTION – B (WRITING SKILLS & GRAMMAR)
II. Grammar
Attempt ANY SEVEN from (4) & (5) combined. (1 × 7 = 7)
4. Fill in the blanks according to the instruction given in the brackets.
a) The streamer __________ yesterday. (Use the correct tense form of the verb ‘sail’)
b) I didn’t know how long she __________ there. (Use the correct tense form of the
verb ‘sit’)
c) My greatest fear is __________. (Fill in the blank with a suitable noun clause)
d) They fought like heroes. (Replace the underlined word with an adverb clause)
5. Do as directed.
a) cultural / there / performances / are / folk dances / regular / including. (Rearrange
the words to form a meaningful sentence)
b) Her looks proclaim her innocence. (Change into a complex sentence)
c) The salesman said, “Are you interested in the scheme?” (Report the given sentence)
d) No other metal is as expensive as gold. (Change into superlative)

5
III. Creative Writing Skills
6. Draft an advertisement on ANY ONE of the following questions. (3 × 1 = 3)
a) You want to purchase a floor. Draft an advertisement in not more than 50 words
for a newspaper detailing your requirement, affordability, and other preferences.
You are Rohit/ Rohini of 12, Shankar Road, New Delhi.
OR
b) On behalf of the principal of your school, draft a classified advertisement in not
more than 50 words inviting application for the posts of teachers lying vacant and
to be filled soon.
7. Design a poster on ANY ONE of the two questions given below. (3 × 1 = 3)
a) As the Director General of Health Services, design a poster to encourage
individuals to step forward for eye donation.
OR
b) Design a poster to be issued by the Delhi Police cautioning people not to touch any
unclaimed object.
8. Write a speech on ANY ONE of the following topics. (5 × 1 = 5)
a) You are the first speaker in the Inter-House Declamation contest being held in your
school. The topic chosen for the contest is ‘Ban Child Labour’. Write your speech
in 120-150 words. You are Ashwini/Anuradha.
OR
b) In the age of technology, mental health has become an escalating issue. Prepare a
speech within 120-150 words on 'Mental Health in the Digital Era,' exploring the
impact of technology on mental well-being and advocating for a harmonious digital
lifestyle. You are Keshav/ Tyasa.
9. Attempt ANY ONE of the following questions on debating. (5 × 1 = 5)
a) “Boarding schools are far better than day schools for the all-around development
of a child.” Express your views either for or against the same in 120-150 words.
OR
b) Students should be involved in decisions regarding school policies and rules
through student councils? Express your views either for or against the motion in
120-150 words.

SECTION – C
10. Read the following extracts and answer the questions that follow. (ANY ONE) (1 × 3 = 3)
i) "The Laburnum top is silent, quite still
In the afternoon yellow September sunlight,
A few leaves yellowing, all its seeds fallen.
Till the goldfinch comes, with a twitching chirrup
A suddenness, a startlement, at a branch end."

1. The Laburnum top, with its leaves yellowing and seeds fallen, symbolizes:
a) Renewal and growth
b) Decline and change
c) Stability and permanence
d) Chaos and confusion

6
2. State whether the given statement is TRUE or FALSE.
The poetic device used in the line, ‘twitching chirrup’ is the same as the one
used in the line, ‘the winter wind wistfully wailed at night’.
3. The Laburnum top in the poem undergoes changes in its surroundings and is
animated by the arrival of the goldfinch. How does the Laburnum top serve
as a metaphor for broader themes explored in the poem?
a) It symbolizes the fleeting nature of beauty and the inevitability of change.
b) It represents the permanence of nature and the constancy of life.
c) It signifies the monotony of everyday existence and the lack of excitement.
d) It conveys the unpredictability of human emotions and relationships.
OR
ii) She’d laugh at the snapshot. “See Betty
And Dolly,” she’d say, “and look how they
Dressed us for the beach.”
1. What emotion does the speaker's mother express when looking at the
snapshot of the two girl cousins paddling?
2. The mother's statement, "See Betty / And Dolly," reflects an attitude of:
a) Mockery
b) Endearment
c) Critique
d) Indifference
3) In the given lines, the mother's laughter serves as a connection between the
speaker's ___________ and her ___________.
a) Past; Present
b) Joy; Sorrow
c) Memory; Reality
d) Family; Friends
11. Read the following extracts and answer the questions that follow. (ANY ONE) (1 × 3 = 3)
i) After months of carefully recording the pharaoh’s funerary treasures, Carter
began investigating his three nested coffins. Opening the first, he found a shroud
adorned with garlands of willow and olive leaves, wild celery, lotus petals, and
cornflowers, the faded evidence of a burial in March or April. When he finally
reached the mummy, though, he ran into trouble.
1. Carter's investigation of the nested coffins is most analogous to:
a) Unveiling a mystery
b) Creating artwork
c) Solving a mathematical problem
d) Designing a structure
2) When Carter "ran into trouble" upon reaching the mummy, it implies that he
encountered:
a) Success
b) Difficulty
c) Joy
d) Surprise

7
3) Complete the sentence appropriately:
The shroud adorned with garlands of willow and olive leaves, wild celery,
lotus petals, and cornflowers suggest that __________.
OR
ii) “Don’t smile smugly. In case you think that it was just my mind playing tricks
and my imagination running amok, look at this.” And, triumphantly, Professor
Gaitonde produced his vital piece of evidence : a page torn out of a book.
Rajendra read the text on the printed page and his face underwent a change.
Gone was the smile and in its place came a grave expression. He was visibly
moved. Gangadharpant pressed home his advantage. “I had inadvertently
slipped the Bakhar in my pocket as I left the library. I discovered my error when
I was paying for my meal. I had intended to return it the next morning.”
1. Fill in the blank with a suitable word.
When Professor Gaitonde produces the page torn out of a book as evidence,
Rajendra's initial smile fades, and he is visibly moved, indicating that the
content on the page has a/an ___________ impact on him.
2. When Gangadharpant says he "inadvertently slipped the Bakhar in my
pocket," the term "inadvertently" suggests that it was done:
a) Deliberately
b) Accidentally
c) Purposefully
d) Intentionally
3. What is ‘Bakhar’?
12. Read the following extracts and answer the questions that follow. (ANY ONE) (1 × 4 = 4)
i) Mourad was considered the natural descendant of this man, although Mourad’s
father was Zorab, who was practical and nothing else. That’s how it was in our
tribe. A man could be the father of his son’s flesh, but that did not mean that he
was also the father of his spirit. The distribution of the various kinds of spirit of
our tribe had been from the beginning capricious and vagrant.
1. According to the passage, what does the distribution of spirit in the tribe
depend on?
a) Practical skills
b) Lineage
c) Capricious and vagrant factors
d) Social status
2. State whether True or False.
Aram’s father, Zorab, was considered the natural descendant of a particular
man in the tribe.
3. What does the statement "Mourad was considered the natural descendant of
this man" suggest about the concept of lineage in the tribe?
a) Lineage is solely based on practical skills.
b) Lineage is determined by one's spiritual qualities.
c) Lineage is capricious and arbitrary.
d) Lineage is determined by the father's social status.

8
4. In the context, what does the term "capricious" most likely mean?
a) Predictable
b) Unchanging
c) Arbitrary
d) Stable
OR
ii) An hour later he went upstairs again, noted the progress made, came down once
more, sat by the kitchen fire. It was still, except for the rustle of a cinder in the
grate and the slow tick-tock of the wall clock. No, there was another sound —the
beat of Morgan’s footsteps as he paced in the street outside. The old woman
opposite him sat in her black dress, quite motionless, her eyes strangely alive and
wise, probing, never leaving his face.
1. What inference can be made about the relationship between the man and the
old woman based on their actions and the described setting?
a) They are engaged in a heated argument.
b) They share a quiet and contemplative companionship.
c) The man is uncomfortable in the woman's presence.
d) The woman is ignoring the man.
2. What can be inferred about Morgan’s state of mind from the given extract?
a) Calm and composed.
b) Anxious and restless
c) Happy and cheerful
d) Indifferent and detached.
3. According to the passage, what is the predominant sound in the described setting?
a) Footsteps
b) Clock ticking
c) Cinders rustling
d) Street noise
4. What does the phrase ‘her eyes strangely alive and wise’ tell you about the
old woman?
13. Answer ANY TWO of the following questions (one from prose and one from poetry)
in 40-50 words. (3 × 2 = 6)
a) In the poem, The Voice of the Rain, what does the phrase ‘strange to tell’ mean?
b) In the poem, ‘Father to Son’, why does the father feel he and his son are like
strangers to each other?
c) What generated the interest of the world in King Tut?
d) How did the narrator and his family prepare for the sea voyage?
14. Answer ANY ONE of the following questions in 40-50 words. (3x1=3)
a) ‘I couldn’t believe what I saw.’ What was so incredible to the speaker? Who is the
speaker here?
OR
b) How did the architect turn the tables on the king himself?

9
I5. Answer ANY ONE of the following questions in 120-150 words. (6 × 1 = 6)
a) Trace the interest of the old grandmother in the education of the author. Do you
think her concern was misplaced?
OR
b) Each photograph is a memory. Justify this statement in the light of the poem, ‘A
Photograph’ by Shirley Toulson.
16. Answer ANY ONE of the following questions in 120-150 words. (6 × 1 = 6)
a) The poem, ‘The Tale of Melon City’, mocks the process of fair trial and proper
judgment. Justify.
OR
b) Justify the title of the story ‘The Address’.

ANSWERS

SECTION – A (READING)
1. Read the passage carefully. (10 )
On the basis of your reading of the passage given above, answer the following questions.
a) i) 270,000
b) ii) I, II, III, IV
c) iii) Jean Henri Dunant
d) iv) Writing a book
e) iii) The Contributions of Jean Henri Dunant
f) iii) I and II
g) time
h) False
i) ii) Cruelties
j) ii) Dedicate.
2. Read the following passage:
Answer the following questions:
a) i) they are the only states to have less than 1,000 people infected with novel
coronavirus
b) iii) Lakshadweep
c) iii) the numbers are rising at a fast pace at rates higher than the national level
d) new/unusal
e) ii) July 31
f) iv) 12.82 lakh
g) iii) three days earlier than Tuesday
h) iii) native

10
3. Read the following passage carefully.
a) Title: Procrastination: Delay things off until the last minute
1. Excuse of Post.
1.1 Enough time to do
1.2. Didn’t learn to work in time
2. Def. of procrast.
2.1. Postponement till tomorrow
2.2. Procrast. derived from Procrastinatus
2.3. Procrast. means delay
3. Expert views of Procrast.
3.1. Steel, psy. calls it”voluntarily delay”
3.2. Dr. Joseph R. Ferrari named “chronic” or “real”
4. Procrast. types
4.1. Stimulation : a thrill from beating a deadline
4.2. Avoiders put off doing things
4.3. decisional procrastinators
5. Results of Procrast.
5.1. low self-esteem
5.2. focus on the past more than the future
5.3. Depends upon deadline
5.4. Procrastinators on avg. got lower grades
6. Types of Procrastinators
6.1. “Passive procrastinators” are more stressed, less efficient
6.2. “Active procrastinators prefer to work under pressure”
Abbreviations:
Procrast.: Procrastination
Def.: Definition
psy.: Psychology
avg. average
b) Summary of the Passage
Procrastination, the act of delaying tasks, is often accompanied by excuses like a
lack of time. Psychologists distinguish between occasional delayers and chronic
procrastinators who may require therapy. The habit can result in low self-esteem,
diminished focus on the present, and academic struggles. Two types, "passive" and
"active" procrastinators, react differently to stress.

SECTION – B (WRITING SKILLS & GRAMMAR)


II. Grammar
Attempt ANY SEVEN from (4) & (5) combined.
4. Fill in the blanks according to the instruction given in the brackets.
a) sailed
b) had been sitting
c) that I will fail the examination
d) as if they were heroes

11
5. Do as directed.
a) There are regular cultural performances, including folk dances.
b) Her looks proclaim that she is innocent.
c) The salesman asked if I was interested in the scheme.
d) Gold is the most expensive metal.
III. Creative Writing Skills
6. Draft an advertisement on ANY ONE of the following questions.
a)
FLOOR REQUIRED
Floor required: North Delhi,
with three bedrooms,
drawing room and a study
room. Ground / first floor
preferred. Price around 30-
35 lakhs. Free hold and
clear from all legal angles.
Owner may contact: Rohit
(99999XXXXX) at 12,
Shankar Road, New Delhi
OR
b)
SITUATION VACANT
DAV Public School, Shastri
Park. Delhi-31 requires 2
PGTs in English against
permanent post. Only
experienced and highly
qualified candidates with
linguistic fluency need to
apply. Salary commensurate
to experience and
professional achievements.
Attend Walk-in interview
with resume, attested
testimonials and passport
size photographs on
Monday 16th May20XX at
9:00am in Principal’s office.
No TA/DA admissible

12
7. Design a poster on ANY ONE of the two questions given below. (3x1=3)
a)

OR
b)

8. Write a speech on ANY ONE of the following topics. (5x1=5)


a) Good morning, Respected Principal, Honourable Judges, Esteemed Teachers, and
Dear Schoolmates,
I, Ashwini, stand before you today with a pressing concern that demands our
immediate attention – the urgent need to ban child labour. Childhood is a sacred
phase meant for education, exploration, and joy. It pains me to acknowledge that,
even today, countless children are deprived of these rights as they are forced into
labour.

13
Banning child labour is not just a legal obligation but a moral imperative. It is our
collective responsibility to ensure that every child enjoys the right to education and
a childhood free from exploitation. Let us unite against this social injustice by
advocating for change, supporting organizations working towards this cause, and
spreading awareness in our communities.
In the journey to ban child labour, we pave the way for a brighter, more
compassionate future. Together, let us be the voice for those who are silenced.
Thank you.
OR
b) Good morning, Respected Principal, Honourable Judges, Esteemed Teachers, and
Dear Schoolmates.
Today, I, Tyasa, will be speaking on the topic, ‘Mental Health in the Digital Era.’
In the era of rapid technological advancement, our connection to the digital realm
is undeniable. However, with the privileges of technology comes a growing
concern for our mental health. The incessant digital stimuli, social media
comparisons, and constant connectivity have ushered in new challenges to our
well-being.
It's crucial to acknowledge the impact of the digital era on mental health and strive
for a balanced approach. Excessive screen time and the constant pursuit of virtual
validation can contribute to stress, anxiety, and feelings of inadequacy. As we
embrace technology, let's prioritize mental well-being by setting boundaries,
practicing digital detox, and fostering genuine human connections.
In this digital age, let us not forget that technology serves as a tool for enrichment
rather than a source of distress. By cultivating a mindful and balanced relationship
with technology, we can pave the way for a mentally healthier and happier society.
9. Attempt ANY ONE of the following questions on debating.
a) Respected judges and my worthy opponents, today, I stand before you all to bring
to light the importance of day schools for the all-round development of a child.
I believe that day schools offer a more balanced and nurturing environment. Boarding
schools, while providing a structured routine and immersive learning, often lack the
warmth and familial support crucial for a child's emotional well-being.
The separation from family at an early age can lead to feelings of isolation and
homesickness, potentially hindering a child's holistic development.
Moreover, day schools allow for a consistent family presence, enabling parents to
actively engage in their child's educational journey. The daily interactions at home
contribute to a well-rounded personality, fostering emotional intelligence and resilience.
In a day school setting, children can strike a healthier balance between academic
pursuits and familial bonds, ensuring a more comprehensive and supportive
educational experience. Overall, the nurturing environment of day schools proves
essential for a child's emotional and social development. Thank you.
OR
b) Respected judges and my worthy opponents, I strongly advocate for students' active
involvement in shaping school policies through student councils.
Empowering students with a role in decision-making fosters a sense of
responsibility, ownership, and civic engagement. Student councils provide a
platform for young minds to voice their concerns, share insights, and contribute to
the establishment of rules that directly affect them.
14
Including students in policy decisions promotes a more inclusive and democratic
educational environment. It encourages critical thinking, leadership skills, and a
deeper understanding of the consequences of rules. Moreover, it nurtures a sense
of community and mutual respect between students and educators.
By involving students in decision-making processes, schools not only acknowledge
the unique perspectives of the youth but also prepare them for active participation
in societal governance. Ultimately, a collaborative approach to policy formation
ensures a more balanced and effective educational system. Thank you.
SECTION – C
10. i) Read the following extracts and answer the questions that follow. (ANY ONE)
1. b) Decline and change
2. FALSE
3. a) It symbolizes the fleeting nature of beauty and the inevitability of change.
OR
ii) 1. Joyful reminiscence
2. b) Endearment
3. c) Memory; Reality
11. Read the following extracts and answer the questions that follow. (ANY ONE)
i) 1. a) Unveiling a mystery
2. b) Difficulty
3. The shroud adorned with garlands of willow and olive leaves, wild celery, lotus
petals, and cornflowers suggest that the burial took place in March or April
OR
ii) 1. emotional
2. b) Accidentally
3. ‘Bakhar’ was a book in Marathi about the Battle of Panipat. The book was
interesting to read as it provided a graphic detail of the Battle.

12. Read the following extracts and answer the questions that follow. (ANY ONE)
i) 1. c) Capricious and vagrant factors
2. False
3. c) Lineage is capricious and arbitrary.
4. c) Arbitrary
OR
ii) 1. b) They share a quiet and contemplative companionship.
2. b) Anxious and restless
3. b) Clock ticking
4. The old woman was wise in experience. She was alert and kept looking at the
doctor as she knew he was inexperienced and feared that he would quit the
case.

15
13. Answer ANY TWO of the following questions (one from prose and one from poetry)
in 40-50 words.
a) The poet’s wonder at the rain’s capacity for communication is conveyed in the
sentence. It is thought that rain cannot communicate like living things. The poet
thinks that the fact that the rain should speak and respond to his query will surprise
and seem strange to the readers as well.
b) Due to lack of engagement between the father and son, the father feels as though
they are like strangers to one another. They do not even have a shared interest
because there is no affection, warmth, or understanding between them. Despite
years of living together, there is a communication gap between them.
c) King Tut was just a teenager when he died. He was the last heir of a powerful
family that had ruled Egypt and its empire for centuries. Since the discovery of his
tomb in 1922, the modem world wondered about what happened to him and
wondered if he could have been murdered.
d) For sixteen years, the narrator and his family spent all their leisure time improving their
seafaring skills in British waters. They bought a boat, Wavewalker, a 23-metre, 30-ton
wooden-hulled vessel that had been professionally built. They spent months fitting it
out and testing it in the roughest weather that they could find.
14. Answer ANY ONE of the following questions in 40-50 words.
a) The speaker in this case is ‘Aram,’ the protagonist of the story.
His cousin brother Mourad had come to invite him for a ride on a magnificent white
horse one fine summer before daybreak. As he did not own the horse, the speaker
was perplexed as to where Mourad had obtained that horse from, as he could not
be a thief.
OR
b) The architect was presented before the king to face conviction. But he reminded
the king of the changes made by the king himself in the building plan. The king
became nervous and said he would consult the wisest man in the state on the issue.
I5. Answer ANY ONE of the following questions in 120-150 words.
a) The elderly grandmother, though lacking formal education herself, ardently
prioritized her grandson's learning. She dutifully prepared his school materials,
accompanied him to school, and assisted with lessons. Despite her unwavering
support, her traditional beliefs clashed with the modern curriculum. The English
language and scientific principles were beyond her comprehension, and the
absence of teachings about God and scriptures left her dissatisfied. The
introduction of music lessons further dismayed her, as she associated it with
societal outcasts.
Regrettably, her concerns were misplaced, as the author, her grandson, absorbed
her moral values while embracing the modern education that equipped him with
English and scientific knowledge. This education proved invaluable in his later
career. The blend of traditional values and progressive education played a big part
in the author's achievements.
OR
b) In Shirley Toulson's "A Photograph," the idea that "Each photograph is a memory"
is beautifully illustrated. The poem centres on a snapshot capturing a fleeting
moment, serving as a potent vessel for memories. Describing the scene of two girl
cousins paddling and the speaker's mother holding hands emphasizes the

16
photograph's role as a visual representation of the past, preserving the essence of
that day.
As the poem unfolds, it explores the transient nature of life. The sea, a backdrop
that remains constant yet ever-changing, symbolizes the enduring and evolving
quality of memories. The speaker's mother revisiting the photograph years later
adds depth to the concept that a photograph encapsulates memories, acting as a
tangible reminder of joy and simplicity.
In summary, Toulson's poem asserts that each photograph is a frozen fragment of
the past, holding a treasure trove of emotions, stories, and the unstoppable march
of time.
16. Answer ANY ONE of the following questions in 120-150 words.
a) "The Tale of Melon City" humorously points out the flaws in fair trial and proper
judgment. The King's rash decisions, like ordering the chief builder's hanging
without a fair inquiry, show the arbitrary nature of justice in the poem. The twist
of the King facing the noose himself adds irony, highlighting the absence of a fair
judicial process. The poem uses humour and exaggeration to criticize authority
figures, suggesting that those in power may act without proper thought. The
appointment of a melon as the new ruler serves as a playful commentary on the
randomness of leadership choices. Overall, the poem satirizes the inadequacies in
governance, showing how authority can be wielded haphazardly, and the
consequences can be absurd.
OR
b) The title "The Address" in Marga Minco's story poignantly underscores the
narrator's wartime experiences and the enduring pain it inflicted. The specific
address, 46 Marconi Street, holds significant memories as the place where the
narrator's mother entrusted family valuables to Mrs. Dorling for safekeeping during
the war. However, upon the narrator's return after the war, the address reveals a
disheartening reality – the precious possessions are found in an unkempt and
unappreciated state, reflecting the shabbiness and pettiness of Mrs. Dorling.
The address, once important for safeguarding cherished items, transforms into a
symbol of disappointment and neglect. The narrator's decision to leave the
possessions behind and forget the address symbolizes a resolution to move forward,
acknowledging the painful repercussions of war. In essence, "The Address"
encapsulates the narrator's journey of confronting loss, disillusionment, and the
imperative of starting anew, leaving the painful past behind.



17
MODEL QUESTION PAPER
FINAL EXAMINATION
SESSION : 2023 - 24
SUBJECT : CHEMISTRY
CLASS - XI
Time Allotted : 3 Hours Maximum Marks : 70

General Instructions
Read the following instructions carefully.
 There are 33 questions in this question paper with internal choice.
 SECTION – A consists of 16 multiple-choice questions carrying 1 mark
each.
 SECTION – B consists of 5 very short answer questions carrying 2
marks each.
 SECTION – C consists of 7 short answer questions carrying 3 marks each.
 SECTION – D consists of 2 case-based questions carrying 4 marks each.
 SECTION – E consists of 3 long answer questions carrying 5 marks each.
 All questions are compulsory.
 Use of log tables and calculators is not allowed
________________________________________________________________
SECTION – A
The following questions are multiple-choice questions with one correct answer. Each
question carries 1 mark. There is no internal choice in this section.
1. Number of atoms of He in 100 u of He (Atomic mass of He is 4 u)
a) 25
b) 50
c) 100
d) 400
2. 6.02 ×1020 molecules of urea are present in 100 mL of its solution. The concentration of
the solution is.
a) 0.02 M
b) 0.01 M
c) 0.001 M
d) 0.1 M
1
3. The number of d-electrons retained in Fe2+ (At. no. of Fe = 26) ion is
a) 3
b) 4
c) 5
d) 6
4. Pauli exclusion principle helps to calculate the maximum number of electrons that can
be accommodated in any.
a) orbital
b) subshell
c) shell
d) All of these
5. The basic strength of the oxides follows the order.
a) Al2O3 > MgO > Na2O
b) Al2O3 < MgO < Na2O
c) Na2O3 < MgO > Al3O2
d) Al2O3 > MgO > Na2O
6. The ionisation enthalpy of nitrogen is more than that of oxygen molecules because of
a) greater attraction of electrons by the nucleus
b) extra stability of the half-filled p-orbitals
c) smaller size of nitrogen
d) more penetrating effect
7. How many σ and π bonds are present in HC≡CC≡CCH 3?
a) 8 and 4 respectively
b) 4 and 8 respectively
c) 6 and 4 respectively
d) 8 and 2 respectively
8. The shape of XeF4 molecule according to VSEPR theory is
a) Square planar
b) Square pyramid
c) Tetrahedral
d) Pyramidal
9. For which of the following reactions would the Ho for the reaction be labelled ∆Hf0?
a) Al(s) + H2 (g) + O2 (g) → Al (OH)3 (s)

b) PCl3 (g) + O2 (g) → POCl3 (g)

c) N2O (g) + O2 (g) → NO (g)


d) CaO (s) + SO2 (g) → CaSO3(s)

2
10. A system suffers an increase in internal energy of 80 J and at the same time has 50 J of
work done on it. What is the heat change of the system?
a) +130 J
b) +30 J
c) –130 J
d) –30 J
11. For the hypothetical reactions, the equilibrium constant (k) values are given.

The equilibrium constant (K) for the reaction A⇌ D is


a) 48
b) 24
c) 12
d) 64
12. The oxidation number of Cr in Cr(CO)6 is —————–
a) 0
b) +2
c) –2
d) +6
13. Given below are two statements labelled as Assertion and Reason
Assertion: Aniline is better nucleophile than anilium ion.
Reason: Anilium ion have +ve charge.
Select the most appropriate answer from the options given below:
a) Both A and R are true and R is the correct explanation of A
b) Both A and R are true but R is not the correct explanation of A.
c) A is true but R is false.
d) A is false but R is true.
14. Given below are two statements labelled as Assertion and Reason
Assertion : Boiling point of alkanes increases with increase in molecular weight.
Reason : van der Waal’s forces increase with increase in molecular weight.
Select the most appropriate answer from the options given below:
a) Both A and R are true and R is the correct explanation of A
b) Both A and R are true but R is not the correct explanation of A.
c) A is true but R is false.
d) A is false but R is true.

3
15. Given below are two statements labelled as Assertion and Reason
Assertion : All the hydrogen atoms in CH2 =C= CH2 are attached to sp2 hybridised
carbon atom.
Reason : All the carbon atoms in its are sp2 hybridized.
Select the most appropriate answer from the options given below:
a) Both A and R are true and R is the correct explanation of A
b) Both A and R are true but R is not the correct explanation of A.
c) A is true but R is false.
d) A is false but R is true.
16. Given below are two statements labelled as Assertion and Reason
Assertion: Alkanes are saturated hydrocarbons.
Reason: Hydrocarbons consist of covalnet bonds.
Select the most appropriate answer from the options given below:
a) Both A and R are true and R is the correct explanation of A
b) Both A and R are true but R is not the correct explanation of A.
c) A is true but R is false.
d) A is false but R is true.

SECTION – B
This section contains 5 questions with internal choice in one question. The following
questions are very short answer type and carry 2 marks each.
17. How many significant figures are present in the following:
i) 4.00005
ii) 0.004
18. Name two intensive and extensive properties of a system.
19. Predict which of the following entropy increases/ decreases?
i) A liquid crystallizes into a solid.
ii) Temperature bf a crystalline solid is raised from 0 K to 115 K.
20. Give two examples each of the groups exerting –I and + I effect when attached to a
chain of carbon atoms.
OR
What is the difference between inductive and electrometric effect?
21. Write chemical equations for combustion reaction of the following hydrocarbons:
i) Butane
ii) Pentene
SECTION – C
This section contains 7 questions with internal choice in one question. The following
questions are short answer type and carry 3 marks each.
22. Dissolving 120 g of urea (Molar mass of urea = 60 g mol-1) in 1000 of water gave a
solution of density 1.15 g/mL. Calculate the molarity of the solution.

4
23. Calculate the energy required for the process
He+ (g) → He2+ (g) + e−
The ionization energy for the H atom in the ground state is 2.18 ×10–18 J. atom–1
24. Calculate the standard enthalpy of formation of CH3OH(l) from the following data:

25. Write the expression for the equilibrium constant for each of the following reactions

26. What is the equilibrium concentration of each of the substances in the equilibrium when
the initial concentration of ICl was 0.78 M?

27. Balance the following equation in basic medium:


C12O7 (g) + H2O2 (1)  ClO2– (aq) + O2 (g)
OR
MnO4–(aq) + S02(g)  Mn2+(aq) +H2S04–(in acidic solution)
What is meant by resonance? Draw the resonating structures of carbonate ion and
explain why all the C–O bond lengths are identical in carbonate ion?
28. Write the IUPAC names of the following:

SECTION – D
The following questions are case-based questions. Each question has an internal
choice and carries 4 (1+1+2) marks each. Read the passage carefully and answer the
questions that follow.
29. “Quantum model of atom”
Orbitals are regions or spaces where there is a maximum probability of finding
electrons. Qualitatively, these orbitals can be distinguished by their size, shape, and
orientation. An orbital of small size means there is more chance of finding the electron
near the nucleus. Shape and orientation mean the direction in which the probability of
finding the electron is maximum. Atomic orbitals can be distinguished by quantum
numbers. Each orbital is designated by three quantum numbers n, l, and ml (magnetic
quantum number) which define energy, shape, and orientation but these are not
sufficient to explain spectra of multi-electrons atoms. Spin quantum number (ms)
determines the spin of electrons. Spin angular momentum of the electron has two
orientations relative to the chosen axis which are distinguished by spin quantum
numbers ms which can take values +1/2 and –1/2.

5
Answer the following questions:
a) How many orbitals are associated with n = 3
b) Describe the orbitals represented by (i) n = 2, l = 1 (ii) n = 4, l = 0.
c) How many radial and angular nodes are present in 2p orbital.
OR
c) What are the four quantum numbers of 19th electron of copper?
30. “Modern periodic table”
Modern periodic table arranges the elements in the increasing order of atomic number.
It has 18 groups and 7 periods. Atomic numbers are consecutive in a period and
increases in the group in a pattern. Elements are divided into four blocks, s-block, p-
block, d-block, and f-block based on their electronic configuration. 78% of elements are
metals, about 20 elements are non-metals, and few elements like B, Si, Ge, As are
metalloids. Metallic character increases down the group but decreases along the period
from left to right. The physical and chemical properties vary periodically with their
atomic numbers. Periodic trends are observed in atomic size, ionization enthalpies,
electron gain enthalpies, electronegativity, and valence. Oxides of metals are basic,
some are amphoteric. Non-metals form acidic oxides, some form neutral oxides. s-
block elements are soft, highly reactive, do not show variable oxidation states. p-block
elements are metals, non-metals as well as metalloids, show variable oxidation states,
exist as solids, liquids, and gases. d-block elements are metals, form coloured ions,
show variable oxidation states, have high melting and boiling points. Lanthanoids and
actinoids are f-block elements, form coloured ions. All actinoids are radioactive.
a) Name two elements which belong to the d-block but are not transition metals.
b) What are representative elements?
c) Which group elements are most electropositive and why?
OR
c) What is the difference between oxidation states of p-block and d-block elements?
SECTION – E
The following questions are long answer type and carry 5 marks each. Two questions
have an internal choice.
31. a) Draw the molecular diagram for Fluorine.
b) State the hybridisation of C in (i) Ethane (ii) Ethyne.
OR
a) Draw the molecular diagram for B2.
b) Explain the structure and hybridisation of Methane
32. i) Identify the functional groups in the following compounds:

6
ii) Explain why alkyl groups act as electron donors when attached to a π-system.
OR
i) For the following bond cleavages, use curved-arrows to show the electron flow
and classify each as homolysis or heterolysis. Identify reactive intermediate
produced as free radical, carbocation and carbanion.

Name a suitable technique of separation of the components from a mixture of


calcium sulphate and camphor.
33. a) Arrange benzene, n-hexane and ethyne in decreasing order of acidic behaviour.
Also give reason for this behaviour.
b) How would you convert the following compounds into benzene?
i) Ethyne
ii) Ethene
OR
An alkene ‘A’ of molecular mass ‘28u’ on treatment with bromine gives a product ‘B’.
The Compound ‘B’ on further dehalogenation with zinc gives back ‘A’. Give the
structures of ‘A’ and ‘B’ and the sequence of reactions.

ANSWER

1. a)
2. d)
3. d)
4. a)
5. b)
6. a)
7. a)
8. a)
9. a)
10. b)
11. b)
12. a)

7
13. a)
14. d)
15. c)
16. a)
17. i) 6
ii) 1
18. Intensive properties: Viscosity, refractive index.
Extensive properties: Mass, volume, heat capacity, etc.
19. i) After crystallization molecules attain an ordered state and therefore entropy
decreases.
ii) When the temperature is raised, a disorder in molecules increases, and therefore
entropy increases.
20. – I group: – CHO, – NO2
+I group: – CH3, – C2H5
OR
Difference between inductive and electromeric effect are as follows:

21. i) +3
ii) –1
iii) –1/2
iv) +6
OR
Oxidation half reaction:
2 K (s) ––→ 2 K+ + 2e-
Reduction half reaction:
C12 (g) 2e- ––→ 2 Cl-
22. n = number of moles of solute = mass/molar mass
V = volume of solution
mass of urea (NH2CONH2) =120g
Molar mass of urea= 60g/mol
Mass of water = 1000g

8
Density of solution. ρ = 1.15 g/mL
n = 120/60 = 2mol
msolution = msolute + msolvent (∵ Law of conservation of mass)
ρsolution × Vsolution = 120 + 1000 (using m = ρ. V)
1.15×Vsolution=1120
Vsolution=974mL=0.974L
thus M=V. n = 0.9742 = 2.05 M
Molarity of the solution is 2.05M
23. Energy associated with Hydrogen like species-

24. On reversing the first equation we get,


CO2(g) + 2H2O (l) → CH3OH (l) + 23O2 (g); ΔrHθ = +726kJmol−1 (on reversing the reaction,
the sign of ΔH also reverses)
The second equation remains as such,
C(graphite) + O2 (g) → CO2 (g); ΔcHθ = −393kJmol−1
On multiplying equation three by 2 we get,
2H2(g) + O2 (g) → 2H2O (l); 2ΔfHθ = 2×−286kJmol−1 = −572kJmol−1
On adding these three equations we get,
C(graphite) + 2H2 (g) + 21O2 (g) → CH3OH (l)
Enthalpy of formation of CH3OH = ΔfH = ΔrHθ + ΔcHθ + 2ΔfHθ
ΔfH = +726 – 393 − 572kJmol−1 = − 239kJmol−1

9
25.

26.

27. Total decrease in oxidation number of Cl2O7 is 8.


Total increase in oxidation number of H2O2 is 2.
H2O2 and O2 are multiplied with 4
Cl2O7 (g) + 4H2O2 (aq) → ClO2− (aq) + 4O2 (g)
Chlorine atoms are balanced
Cl2O7 (g) + 4H2O2 (aq) → 2ClO2− (aq) + 4O2 (g)
O atoms are balanced by adding 3 water molecules.
Cl2O7 (g) + 4H2O2 (aq) → 2ClO2− (aq) + 4O2 (g) + 3H2O (l)
H atoms are balanced by adding 2 hydroxide ions and 2 water molecules.
CI2O7 (g) + 4H2O2 (aq) + 2OH− → CIO2− (aq) + 4O2 (g) + 5H2O (l)
OR
The balanced half reaction equations are:
Oxidation half equation:
S02(g) + 2H2O(l)  HS04– (aq) + 3H+(aq) +2e– …(i)
Reduction half equation:
MnO4–(aq) + 8H+(aq) + 5e–  Mn2+(aq) + 4H2O(l) ………..(ii)
Multiply Eq. (i) by 3 and Eq. (ii) by 2 and add, we have,
2MnO4–(aq) + 5S02(g) + 2H20(l) + H+(aq)  2Mn2+(aq) + 5HSO4–(aq)

10
28. a) 2- Methylbut-2-ene
b) Pent-1-en-3-yne
c) Buta-1, 3-diene.
29. a) 9
b) i) 2p
ii) 4s
c) Radial nodes = 0, Angular nodes = 1
OR
c) n = 3, l = 2, m = -2, s = ½
30. a) Zn, Cd
b) s-block and p-block elements are called representative elements or main group
elements.
c) Group 1 due to the largest atomic size and lowest ionization enthalpies in
respective periods.
OR
c) In p-block elements, oxidation states differ by 2 and the lower oxidation state is
more stable whereas in d-block, oxidation state differ by 1 and mostly higher
oxidation state is more stable.
31. a)

b) i) sp3
ii) sp2

11
OR
a)

b) The Carbon in methane has the electron configuration of 1s22s22p2. According to


Valence Bond Theory, the electrons found in the outermost (valence) shell are the
ones we will use for bonding overlaps. This will be the 2s and 2p electrons for
carbon.
As you know, p electrons are of higher energy than s electrons. This means that
the two p electrons will make shorter, stronger bonds than the two s electrons
right? But this is not what we see. We see a methane with four equal length and
strength bonds. So how do we explain this? Simple: Hybridization

One of the s orbital electrons is promoted to the open p orbital slot in the carbon
electron configuration and then all four of the orbitals become "hybridized" to a
uniform energy level as 1s + 3p = 4 sp3 hybrid orbitals.

12
32. i)

ii) Due to hyperconjugation, alkyl groups act as electron donors when attached to a
π- system as shown below:

OR

i) Sublimation. Because camphor can sublime whereas CaSO4 does not.

13
33. a) Acidic character of species is defined on the basis of ease with which it can lose
its H-atoms. The hybridization state of carbon in the given compound is:

As the s-character increases, the electronegativity of carbon atom increases, and


the electrons of C-H bond pair lie closer to the carbon atom. As a result, partial
positive charge of H- atom increases and H+ ions are set free. The s-character
increases in the order:
sp3 < sp2 < sp
Thus, the decreasing order of acidic behaviour is Ethyne > Benzene > Hexane.
b) i) Ethyne to benzene

ii) Ethene to benzene

OR
A + Br2 → B
(alkene)
Mass=28u
CH2=CH2 + Br2 → CH2Br-CH2Br
(A) (B)

CH2Br−CH2Br + Zn → CH2=CH2 + ZnBr2


(B) (A)



14
MODEL QUESTION PAPER
FINAL EXAMINATION
SESSION : 2023 - 24
SUBJECT : PHYSICS
CLASS - XI
Time Allotted : 3 Hours Maximum Marks : 70
General Instructions:
 There are 33 questions in all. All questions are compulsory.
 This question paper has five sections: Section A, Section B, Section C,
Section D and Section E.
 All the sections are compulsory.
 Section A contains sixteen questions, twelve MCQ and four assertion
reasoning based of 1 mark each, Section B contains five questions of 2
marks each, Section C contains seven questions of 3 marks each, Section
D contains two case study-based questions of 4 marks each and Section E
contains three long questions of 5 marks each.
 There is no overall choice. However, an internal choice has been provided
in one question in Section B, one question in Section C, one question in
each CBQ in Section D and all three questions in Section E. You have to
attempt only one of the choices in such questions.
 Use of calculators is not allowed.
________________________________________________________________
SECTION – A
1. If we do an experiment by swinging a small ball by a thread of length 100 cm, what will
be the approximate time for complete to and fro periodic motion?
a) 4 sec
b) 2 sec
c) 6 sec
d) 1 sec (1)
2. Water waves produced by a motorboat sailing in water are
a) Neither longitudinal nor transverse
b) Both longitudinal and transverse
c) Only longitudinal
d) Only transverse (1)

1
3. If the force and displacement of particle in the direction of force are doubled, then the
work done would be
a) Double
b) 4 times
c) half
d) ¼ times (1)
4. Surface tension is due to
a) frictional forces between the molecules
b) cohesive forces between the molecules
c) adhesive forces between the molecules
d) both (b) and (c) (1)
5. On a hilly region, water boils at 95°C. The temperature expressed in Fahrenheit is
a) 100°F
b) 20.3°F
c) 150°F
d) 203°F (1)
6. A molecule lying along a straight line possess degree of freedom.
a) One
b) Two
c) Three
d) Four (1)
7. According to Carnot, which type of engine working between two temperatures T1 and T2
have maximum efficiency?
a) Reversible engine
b) Irreversible engine
c) External combustion engine
d) Diesel engine. (1)
8. A spring is stretched by applying a load to its free end. The strain produced in the spring is
a) Volumetric
b) Shear
c) Longitudinal and shear
d) Longitudinal (1)
9. The ratio of the magnitude of potential energy and kinetic energy of a satellite is
a) 1:2
b) 2:1
c) 3:1
d) 1:3 (1)
10. In pure rotation,
a) All particles of the body move in a straight line
b) All particles of the body move in concentric circles
c) All particles of the body move in non-concentric circles
d) All particles of the body have same speed (1)
2
11. A spring (spring constant= k) is cut into 4 equal parts and two parts are connected in
parallel. What is the effective spring constant?
a) 4k
b) 16 k
c) 8k
d) 6k (1)
12. Two sound waves of slightly different frequencies propagating in the same direction
produce beats due to
a) Interference
b) Diffraction
c) Reflection
d) Refraction (1)
From question 13 to 16 are assertion reasoning based questions. Two statements are
given-one labelled Assertion (A) and the other labelled Reason (R). Select the correct
answer to these questions from the codes (a), (b), (c) and (d) as given below. a) Both A
and R are true and R is the correct explanation of A b) Both A and R are true and R is
NOT the correct explanation of A (c) A is true but R is false (d) both A and R are false
13. Assertion : Velocity of particles, while crossing mean position (in stationary waves) varies
from maximum at antinodes to zero at nodes.
Reason : Amplitude of vibration at antinodes is maximum and at nodes, the amplitude is
zero, And all particles between two successive nodes cross the mean position together. (1)
14. Assertion : Simple harmonic motion is a uniform motion.
Reason : Simple harmonic motion is not the projection of uniform circular motion. (1)
15. Assertion : The heat supplied to a system is always equal to the increase in its internal
energy.
Reason : When a system changes from one thermal equilibrium to another, some heat is
absorbed by it. (1)
16. Assertion : Specific heat capacity is the cause of formation of land and sea breeze.
Reason : The specific heat of water is more than land. (1)
SECTION – B
17. Define Poisson’s ratio. Derive an expression for Poisson’s ratio. (2)
OR
Distinguish between laminar flow and turbulent flow. Define critical velocity. (2)
18. If the displacement of two waves at a point is given by:

Calculate the resultant amplitude? (2)

3
19. Two bodies of masses 10 kg and 20 kg respectively kept on a smooth, horizontal surface
are tied to the ends of a light string. A horizontal force F=600 N is applied to (i) B (ii) A
along the direction of string. What is the tension in the string in each case?

(2)
-1
20. A spring gun has a spring constant of 80 N cm . The spring is compressed 12 cm by a
ball of mass 15 g. How much is the potential energy of the spring? If the trigger is pulled,
what will the velocity of the ball be? (2)
21. Four particles of masses m, m,2m and 2m are placed at four corners of a square of side
‘a’. Find the centre of mass of the system. (2)
SECTION – C
22. a) An iron ring is attached to the rim of a wooden wheel of a bullock cart by a
blacksmith. The diameter of the rim is 5.243m and the diameter of the iron ring is
5.231m at 37 degrees Celsius. What should be the temperature of the ring to fit the
rim of the wheel?
b) State the difference among conduction, convection, and radiation. (3)
23. A metre-long tube opens at one end, with a movable piston at the other end, shows
resonance with a fixed frequency source (a turning fork of frequency 340 Hz) when the
tube length is 25.5 cm or 79.3 cm. Estimate the speed of sound in air at the temperature
of the experiment. The edge effect may be neglected. (3)
OR
A pipe 20 cm long is closed at one end. Which harmonic mode of the pipe is resonantly
excited by a 430 Hz source? Will the same source be in resonance with the pipe if both
ends are open? (speed of sound in air is 340 ms-1). (3)
24. Prove Archimede’s principal mathematically. (3)
25. a) Two wires made of same material are subjected to forces in the ratio 1 : 4. Their
lengths are in the ratio 2 : 1 and diameters in the ratio 1 : 3. What is the ratio of
their extensions ?
b) Stress and pressure are both forces per unit area. Then, in what respect does stress
differ from pressure? (3)
26. a) If radius of earth is 6400km, what will be the weight of 1 quintal body if taken to
the height of 1600 km above the sea level?

4
b) State two essential requisites of geostationary satellite? (3)
27. a) A car weighs 1800 kg. The distance between its front and back axles is 1.8 m. Its
centre of gravity is 1.05 m behind the front axle. Determine the force exerted by
the level ground on each front wheel and each back wheel.
b) Give the location of the centre of mass of a (i) sphere, (ii) cylinder, (iii) ring, and
(iv) cube, each of uniform mass density. Does the centre of mass of a body
necessarily lie inside the body? (3)
28. a) When we try to close a water tap with our fingers, fast gets of water gush through
the openings between our fingers. Explain why?
b) Torricelli’s barometer used mercury. Pascal duplicated it using French wire of
density 984.Determine the height of the wire column for normal atmospheric
pressure. (3)

SECTION – D
29. Bernoulli’s principle formulated by Daniel Bernoulli states that as the speed of a moving
fluid increases (liquid or gas), the pressure within the fluid decreases.
Although Bernoulli deduced the law, it was Leonhard Euler who
derived Bernoulli’s equation in its usual form in the year 1752.
i) Bernoulli’s theorem is based on conservation of
a) linear momentum
b) mass
c) energy
d) angular momentum (1)
ii) Bernoulli’s theorem is applicable only for
a) Turbulent flow
b) Viscous flow
c) Steady flow
d) Non viscous and incompressible flow (1)
iii) An incompressible fluid flows steadily through a cylindrical pipe which has radius
2R at point A and R at a point B further along the flow direction. If the velocity at
A is v, then that at B is:
a) v/2
b) v
c) 2v
d) 4v (1)
iv) Water flows along a horizontal pipe whose cross-section is not constant. The
pressure is 1 cm of Hg, where the velocity is 35 cm/s. At a point where the velocity
is 65 cm/s, the pressure will be
a) 0.89 cm of Hg
b) 8.9 cm of Hg
c) 0.5 cm of Hg
d) 1 cm of Hg (1)

5
OR
v) A straight or curved path, such that tangent to it at any point gives the direction of
flow of liquid at that point is known as
a) Turbulent flow
b) Terminal flow
c) Random flow
d) Streamline flow (1)
30. When any object is thrown from horizontal at an angle θ except 90°, then the path
followed by it is called trajectory, the object is called projectile and its motion is called
projectile motion.
If any object is thrown with velocity u, making an angle θ, from horizontal, then

 Horizontal component of initial velocity = u cos θ.


 Vertical component of initial velocity = u sin θ.
 Horizontal component of velocity (u cos θ) remains same during the whole journey
as no acceleration is acting horizontally.
 Vertical component of velocity (u sin θ) decreases gradually and becomes zero at
highest point of the path.
i) A ball is projected upward at a certain angle with the horizontal. which of the
following statement is/are correct? At highest point
a) velocity of the projectile is not zero
b) acceleration of the projectile is zero
c) velocity of the projectile is along the horizontal direction
d) Acceleration of the projectile is vertically upwards (1)
ii) Which of the following remains constant during the motion of the projectile fired
from a planet,
a) Kinetic energy
b) Momentum
c) Vertical component of the velocity
d) Horizontal component of the velocity (1)
iii) A stone is just released from the window of a train moving along a horizontal
straight track. The stone will hit the ground following
a) Hyperbolic path
b) Straight path
c) Circular path
d) Parabolic path (1)

6
iv) The maximum range of gun on horizontal terrain is 16 km. If g = 10 m/s2. What
must be the muzzle velocity of the shell?
a) 400 m/s
b) 100 m/s
c) 200 m/s
d) 50 m/s (1)
OR
v) A ball is projected with velocity 10 m/sec at angle of 30° with the horizontal
surface. The speed of the ball after 1 second will be (Use g=10m/sec2)
a) 5 m/sec
b) 20 m/sec
c) 8 m/sec
d) 10 m/sec (1)
SECTION – E
31. A body of mass m is attached to one end of a massless string which is suspended
vertically from a fixed point. The mass is held in hand so that the spring is neither
stretched nor compressed. Suddenly the support of the hand is removed. The lowest
position attained by the mass during oscillation is 4 cm below the point, where it was
held in hand.
a) What is the amplitude of oscillation?
b) Find the frequency of oscillation. (5)
OR
a) What is Simple pendulum? Find an expression for the time period and frequency
of a simple pendulum?

b) A particle is in linear simple harmonic motion between two points, A and B, 10 cm


apart. Take the direction from A to B as the positive direction and give the signs of
velocity, acceleration and force on the particle when it is
i) at the end A,
ii) at the end B. (5)
32. i) The given progressive wave y = 5 sin (100 t – 0.4 x) where x, y is in m, and t is
in seconds, what is the
a) amplitude
b) wavelength
c) frequency

7
d) wave velocity
e) particle velocity amplitude? (5)
OR
I) State the difference between stationary waves and progressive waves.
II) A steel sod 100 cm long is clamped at its middle. The fundamental frequency of
longitudinal vibrations of the rod as given to be 2.53 kHz. What is the speed of
sound in steel? (5)
33. Two discs of moments of inertia I1 and I2 about their respective axes (normal to the disc
and passing through the centre), and rotating with angular speed ω1 and ω2 are brought
into contact face to face with their axes of rotation coincident.
a) Does the law of conservation of angular momentum apply to the situation? why?
b) Find the angular speed of the two-disc system.
c) Calculate the loss in kinetic energy of the system in the process.
d) Account for this loss. (5)
OR
a) Why is moment of inertia called rotational inertia?
b) Calculate M.I of a uniform circular disc of mass 500gm and radius 10cm about
(i) Diameter (ii) axis tangent to the disc and parallel to diameter
c) Axis passing through centre and perpendicular to its plane? (5)

ANSWER
1. b) 2 sec
2. b) both
3. b) 4 times
4. d) both cohesive as well adhesive.
5. d) 2030F
6. a) one
7. a) Reversible engine
8. b) shear
9. b) 2:1
10. c) All particles of the body move in non-concentric circles
11. b) 16 k
12. a) interference
13. a)
14. d)
15. d)
16. a)

8
17. a) It is defined as the ratio of the change in lateral width per unit width to change in
axial length per unit length caused by the axial stretching or stressing of a material.

OR
Laminar flow is quiet, slow-moving water characterized by water particles moving in a
straight direct line within parallel layers. Turbulent flow, on the other hand, is louder,
faster moving water characterized by erratically moving water particles that mix between
the parallel layers.
Critical velocity is defined as the speed at which a falling object reaches when both
gravity and air resistance are equalized on the object. The other way of defining critical
velocity is the speed and direction at which the fluid can flow through a conduit without
becoming turbulent.
Let the two waves are:
y₁ = a₁ sin ωt and y₂ = a₂ sin (ωt + φ)
Where,

9
X(CM)= a/2, Y(CM)=2a/3
a)

10
In conduction, heat transfer occurs between objects by direct contact. In convection, the
heat transfer takes within the fluid. In radiation, heat transfer occurs through
electromagnetic waves without involving particles. The heat transfer takes place due to
the difference in temperature.

Now, (2n-1)υ/4l1 = 340. Substituting values


(2 × 1 –1) υ × 100/4 × 25.5 = 340 or υ =346.8 ms–1

11
b) Stress can be defined as the internal resistive force to the deformation per unit
area. Pressure can be defined as the amount of force applied per unit area.
Due to stress, the pressure will not be developed. Due to pressure, stress will be
developed. Stress can be either a positive or a negative force. The pressure is
always a positive force.
26. a) R = 6400km = 6400 x 103m
h = 1600km
w = mg = 1 quintal = 100 kg = 100x9.8 N
weight (w) = mgh

w = mg

w = 100×9.8

w = 64×9.8N = 64kg
b) (1) The period of revolution of a satellite around the earth should be same as that
of earth about its own axis (T=24hrs)

12
(2) The sense of rotation of satellite should be same as that of the earth about its
own axis i.e. from west to east in anti-clockwise direction
27. a) Let F1 and F2 be the forces exerted by the level ground on front wheels and back
wheels respectively.
Considering rotational equilibrium about the front wheels,
F2 × 1.8 = mg × 1.05 or F2 = 1.05/1.8 × 1800 × 9.8 N = 10290 N Force on each
back wheel is =10290/2 N or 5145 N.
Considering rotational equilibrium about the back wheels.
F1 x 1.8 = mg (1.8 – 1.05) = 0.75 x 1800 x 9.8
or F1=0.75 x 1800 x 9.8/1.8 = 7350 N
Force on each front wheel is 7350/2 N or 3675 N.
b) In all the four cases, as the mass density is uniform, centre of mass is located at
their respective geometrical centres.
No, it is not necessary that the centre of mass of a body should lie on the body. For
example, in case of a circular ring, centre of mass is at the centre of the ring, where
there is no mass.
28. a) According to equation of continuity, velocity is inversely proportional to the area
of cross section. Since area between two fingers is very small as compared to the
area of water tap above it, hence fast gets of water gush out through openings
between the fingers.
b) pressure exerted by h height of wine column= pressure exerted by 76 cm of Hg column
h*984*9.8=0.76*13.6*1000*9.8
h=10.5 m
29. i) c) energy
ii) d) Non viscous and incompressible flow
iii) d) 4v
iv) a) 0.89 cm of Hg
v) Streamline flow
30. I) C
II) D
III) d
IV) a
V) d
31. a) when mass m is held in support by hand the extension in spring will be zero as no
deforming force acts on spring.
Let the mass reaches at its new position unit displacement from previous.
Then P.E. of spring or mass = gravitational P.E. lost by man

13
But P.E. due to spring is

Mean position of spring by block will be when let extension is then

From and

The amplitude of oscillator is the maximum distance from mean position

b) Time Period which does not depend on amplitude

or

14
Oscillator will not rise above the positive from where it was released because total
extension in spring is 4cm when released and amplitude is 2 cm. So it oscillates
below the released position.
OR
A simple pendulum is the most common example of the body executing S.H.M, it
consist of heavy point mass body suspended by a weightless inextensible and
perfectly flexible string from a rigid support, which is free to oscillate.
Let m = mass of bob
l = length of pendulum
Let O is the equilibrium position, OP = X
Let θ = small angle through which the bob is displaced.
The forces acting on the bob are:-
1) The weight = M g acting vertically downwards.
2) The tension = T in string acting along Ps.
Resolving Mg into 2 components as Mg Cos θ and Mg Sin θ,
Now, T = Mg Cos θ
Restoring force F = – Mg Sin θ
– ve sign shows force is directed towards mean position.

Let θ = Small, so Sin θ ≈ θ =

Hence F = - mg θ
F = – mg

Now, In S.H.M, F = k x →4) k = Spring constant


Equating equation3) & 4) for F
–kx=–mg

Spring factor = k =

Inertia factor = Mass of bob = m


Now, Time period = T

b) a) Zero, Positive, Positive


b) Zero, Negative, Negative

15
32. i) The given equation is y=5sin(100πt−0.4πx)y=5sin(100πt-0.4πx)
Compart is with the standard form of eqn. of progressive wave :
y = r sin [2πtT−2πλ x]
y = r sin [2πtT-2πλ x]
a) amplitude, r=5mr=5m
b) 2πλ=0.4π,
λ=2π0.4π=5m
2πλ=0.4π,
λ=2π0.4π=5m
c) 2π/T=100π,1/T=v=100π/2π=50Hz
d) wave velocity, υ=vλ=50×5=250m/sυ=vλ=50×5=250m/s
e) particle velocity, dy/dt=5×100πcos(100πt−0.4πx)dydt=5×100πcos(100πt-
0.4πx) (dy/dt)max=500π×1m/s=500πm/s(dy/dt)max=500π×1m/s=500πm/s
OR
Stationary waves are still, they do not travel in a medium. Progressive waves travel in
the medium in different phases. Stationary waves are at rest but they perform simple
harmonic motion (SHM) in varying amplitude. Progressive waves, on the other hand,
perform simple harmonic motion in its mean position.
Given: =2.53kHz = 2.53 × 103Hz (L) Length of steel rod = 100 cm = 1m.
when the steel rod clamped at its middle executes longitudinal vibrations of its
fundamental frequency, then or λ = 2L = 2 × 1 = 2m. The speed of sound in steel

33. a) The law of conservation of angular momentum can be apply to this situation
because there is no net external torque on the system. Gravitational and its normal
reaction are external forces but their net torque is zero, hence will not produce any
effect.
b) By the law of conservation of angular momentum

Where I and are the moment of inertia and angular speed of combined system.

c) Final kinetic energy = (rotational +translational) Kinetic energy

AS there is no translational energy

16
As no translational motion in the discs so and are zero.

d) Negative sign shows that as the energy is lost during friction between the
moving surface of discs.
OR
a) Moment of inertia is called rotational inertia because it measures moment of inertia
during its rotational motion.
b) i)

ii)

17
iii)

     

18
MODEL QUESTION PAPER
FINAL EXAMINATION
SESSION : 2023 - 24
SUBJECT : COMPUTER SCIENCE
CLASS - XI
Time Allotted : 3 Hours Maximum Marks : 70
General Instructions
 Please check this question paper contains 35 questions.
 The paper is divided into 5 Sections- A, B, C, D and E.
 Section A, consists of 18 questions (1 to 18). Each question carries 1 Mark.
 Section B, consists of 7 questions (19 to 25). Each question carries 2 Marks.
 Section C, consists of 5 questions (26 to 30). Each question carries 3 Marks.
 Section D, consists of 2 questions (31 to 32). Each question carries 4 Marks.
 Section E, consists of 3 questions (33 to 35). Each question carries 5 Marks.
 All programming questions are to be answered using Python Language only.
________________________________________________________________
Section – A
1. A collection of 8 bits is called a ____________. (1)
2. The validity of the copyright is for ____________. (1)
a) Throughout the life time of the author.
b) Throughout the life time of the author plus 20 years after his/her death
c) Throughout the life time of the author plus 40 years after his/her death.
d) Throughout the life time of the author plus 60 years after his/her death.
3. Which of the following falls under Utility Software’s? (1)
a) Text Editor
b) Back up
c) Disc defragmenter
d) All of these.
4. Write the type of error returned by the following statement. (1)
print(eval(13))
a) Syntax Error
b) Type Error
c) Semantic Error
d) Run Time Error
1
5. Choose the correct output: (1)
s = 'Computer Program'
print(s[-10:-3])
a) er Prog
b) ter Pro
c) er Progra
d) Computer
6. Which of the following statement will be used to convert the string elements into
lowercase? (1)
a) print(S.lowercase())
b) print(S.changelower()
c) print(S.lower())
d) print(lower())
[S is the name of the string variable]
7. Choose the correct answer of the following conversion? (1)
(1001 . 11001)2
a) (A . C8)16
b) (9 . C8)16
c) (A . D7)16
d) (9 . D7)16
8. Given the values for the following variables, what will be the output of the following? (1)
a = 75
b = 96
c = 36
print(a - c // 5 > 60 != c + 7 == b+1) # Statement 1
print((a + b) / 2 > (b - c) ** 2) # Statement 2
a) Both Statement 1 and 2 are True
b) Both Statement 1 and 2 are False
c) Statement 1 is True but Statement 2 is False
d) Statement 2 is True but Statement 1 is False
9. What is the result of the following code? (1)
D1 = { 'abc' : 5 , 'def' : 6 , 'ghi' : 7 }
print(D1[0])
a) abc
b) 5
c) [‘abc’ : 5]
d) Error

2
10. What will be the output of the following code? (1)
import random
day_list['m','t','w','th','f','sa']
low = random.randint(1,4)
high = random.randint(2,5)
for num in range(low,high):
print(day_list[num],end="*")
a) m*w*th*f*
b) m*t*w*th*
c) t*w*th*
d) t*w*th*sa*
11. Evaluate the following expression and identify the correct answer. (1)
7 - 2 + 3 ** 2 + (68 - 2) // 10
a) 70
b) 70.6
c) 20.6
d) 20
12. You received an email purportedly from Flipkart regarding a discount offer for a
product with a link provided. What should you do about it? (1)
a) Click on the link immediately so that you do not miss out on the opportunity.
b) Verify the display name of the sender and proceed to click on the link if it looks
genuine.
c) Verify the email id of the sender and proceed to click on the link if it looks genuine
d) Go directly to Flipkart website and look for the offer there.
13. Predict the output of the following code: (1)
l1 = [19, 29, 81, 63, 42]
l2 = []
for i in l1:
if i % 3 == 0:
l2.append(i)
print(max(l2))
a) 81
b) 19
c) 29
d) 63
14. Which of the following will raise an error if the given dictionary is empty? (1)
a) del statement
b) pop ( )
c) popitem ( )
d) All of these
15. VIRUS is an acronym used for ________________________. (1)

3
16. Consider the loop given below. What will be the final value of i after the loop? (1)
for i in range(10):
break
a) 10
b) 0
c) 9
d) Error
Q17 and 18 are ASSERTION AND REASONING based questions. Mark the correct
choice as
a) Both A and R are true and R is the correct explanation for A
b) Both A and R are true and R is not the correct explanation for A
c) A is True but R is False
d) A is False but R is True
17. Assertion (A) : Arithmetic operators are used to perform operations like addition,
subtraction, comparison, division and multiplication.
Reasoning (R) : The output of Arithmetic operators is always a value. (1)
18. Assertion (A) : You can modify a tuple using packing and unpacking method.
Reasoning (R) : Consider the given example to modify the element 1.5 to 100 in a
tuple. (1)
item = ('apple', 'seed', 1.5, 9)
temp = list(item)
temp[2]=100
item = tuple(temp)
print(item)
('apple', 'seed', 100, 9)
Section – B
19. (1 + 1 = 2)
a) What is meant by Active Digital Footprints?
b) What is a Free Software?
OR
a) What is meant by Passive Digital Footprints?
b) What is a Free and Open Source Software?
20. Rewrite the following code after removing all errors (if any). Underline each correction
done in the code. (2)
x = integer(input("Enter a value"))
if x%2=0:
print(x, is even)
else;
print(x, 'is odd)

4
21. Write a program in Python that counts the number of strings where the string length is
less than 3 from a list of strings.
Sample List: ['ab','cde','fg','h','ijk']
Expected result: 3 (2)
OR
Write a Python program that accepts a string from the user and counts the number of
vowels present in it.
If the sentence is: roses are red
The output should be: 5
22. Draw the logic circuit diagram for the following Boolean Expression. (2)
F = (A + B) . (B + C)
23. Write the Python statement for each of the following task using BUILT – IN functions /
methods only. (1 + 1 = 2)
i) To add the value 100 at the 5th index location of the list A, where
A=[1,2,3,4,5,6,7]
ii) To remove the key 3 from a dictionary A, where A={1:100 ,2:200, 3:300, 4:400}
OR
i) To get the index position of the value 3 of the list A, where A = [1,2,3,4,5]
ii) To join ‘@’ sign after each character of the string x, where x = 'pass'
24. What is Ransomware? (2)
OR
What is Eavesdropping?
25. Differentiate between Hardware and Software. (2)
Section – C
26. Predict the output of the following Python code : (3)
n = "r0sE is rED" # 0 is a digit
r=''
for i in n:
if i.isupper():
r += '@'
elif i.islower():
r += '$' + n[1:3]
elif i.isdigit():
r += "^" +n[3:5]
else:
r+='#'
print(r)
27. State the Absorption Laws and prove them using truth tables. (Both the laws) (3)

5
28. Write a program to input names of n students and store the in a tuple. Also, input a name
from the user and find if the student is present in the tuple or not. (3)
OR
Write a python program to create a dictionary with name as key and roll number and
marks as values. Finally display the names of students who have marks above 60.
29. Sita and Gita are two friends who are studying online, for which they open various
websites. They noticed that someone is sending a barrage of online messages and emails
to them by observing each activity of theirs on internet. (1 × 3 = 3)
i) What would you call this act of the ill – intentioned person?
ii) What do you think what kind of software is designed by this ill – intentioned person
to enter their device or computer system to gather all information, observe all user
activities, and send this information to third parties.
iii) One of their friend suggested to install a software program or a hardware device
that will filter all data packets coming through the internet or a network. What is it
known as?
30. Write a python program to print the following pattern: (3)
*
* *
* * *
* * * *
* * * * *
* * * *
* * *
* *
*
Section – D
31. Sarita works in a telecom office, where she has to prepare bills for her customers based
on the following details.
First 50 calls are free.
Next 150 calls Rs1/call.
Next 200 calls Rs4/call.
Next 100 Rs6/call.
Above 500, Rs8/call.
GST is charged at a flat rate of 6% for the bill.
Write a python program for her. (4)
32. Tridib was told by his computer teacher to write a python program for the following
series: (4)
1+2 1+2+3 1+2+3+⋯… +
+ + ⋯………………+
1 ×2 1 ×2 ×3 1 ×2 ×3 ×…… ×
Help him write the code.

6
Section – E
33. Mr. Abhay is the administrator of a school computer network. Help him to take
appropriate decisions to achieve computer security and safe online access. (5)
a) Suggest a device to be used to filter network traffic to / from school network to
provide network security.
b) Which software can he use to get protection from viruses and spyware?
c) What should he disable in the browser so that online patterns are not tracked?
d) Suggest an active protection mechanics to block spams.
e) Suggest an active protection solution to PC intrusion.
34. (½ × 4 +3 = 5)
a) Consider the following string:
x = 'Have faith in God'
What will be the output of the following operations:
(i) print(x[-7:-1])
(ii) print(x[::2])
(iii) print(x.index('in'))
(iv) print(x.replace("God","Supreme Power"))
b) Write a Python program that reads a line, then counts how many times a substring
‘is’ appears in the line and displays the count.
OR
a) Consider the following string:
x = 'Have faith in God'
What will be the output of the following operations:
(i) print(x[-2::])
(ii) print(x[::-2])
(iii) print(x[-1:-7])
(iv) print(x.split('i'))
b) Write a Python program that reads a string and then prints a string that capitalizes
every other letter in the string.
Sample input – school
Output – sChOoL
35. (1 + 4 = 5)
a) Explain the count ( ) function of list.
b) Write Python program to display square of an element if it is an integer and change
the caser if it is a string.
Sample List L = [10,'FUN',4,'HAPPY',90,'SAD',37,'ENJOY']
OR
a) Explain fromkeys ( ) function of dictionaries.

7
b) Write Python program to add 5 to all odd values and 10 in all even values of a
list. Also display the list
Input L = [1,2,3,4,5,6,7,8,9,10]
Output L = [6, 12, 8, 14, 10, 16, 12, 18, 14, 20]

ANSWER
Section – A
1. byte
2. d) Throughout the life time of the author plus 60 years after his / her death.
3. d) All of these.
4. b) Type Error
5. a) er Prog
6. c) print(S.lower())
7. b) (9 . C8)16
8. b) Both Statement 1 and 2 are False
9. d) Error
10. c) t*w*th*
11. d) 20
12. d) Go directly to Flipkart website and look for the offer.
13. a) 81
14. c) popitem ( )
15. Vital Resource Under Seize.
16. b) 0
17. d) A is False but R is True.
18. b) Both A and R are True and R is not the correct explanation for A
Section – B
19. a) Active Digital Footprints include data that we are intentionally submit online.
This includes emails we write, responses or posts we make on different websites
or mobile apps etc.
b) Free software are those for which there is no license fee, it can be downloaded
and shared without any restrictions but their source code is not available.
OR
a) Passive Digital Software : They are the digital data trail we leave online
unintentionally is called Passive Digital Footprints. This include the data
generated when we visit a website, use a mobile app, browse the internet etc.
b) Free and Open Source Software allow users to not only access but also modify (or
improve) them as their source code is also available.

8
20.
x = integer(input("Enter a value")) # not integer must be int
if x%2=0: # = missing
print(x, 'is even')
else; # ; should be replaced with :
print(x, 'is odd) # closing ' mark missing
21.
li = ['ab','cde','fg','h','ijk']
c=0
for i in li:
if len(i)<3:
c+=1
print(c)
OR
x = input("enter a string")
c=0
for i in x:
if i in ['A', 'a', 'E', 'e', 'I', 'i', 'O', 'o', 'U', 'u'] :
c +=1
print(c)
22.

23.
i) A.insert(5,100)
ii) A.pop(3)
OR
i) A.index(3)
ii) "@".join(x)
24.
Ransomware is a type of virus that locks your computer and refuses to let you access
information unless you pay a ransom. It is a huge hazard to both individuals and
businesses. Ransomware encompasses virus and Trojan attacks that can wipe out files
and the entire system.
OR

9
Eavesdropping is a passive attack in which the attacker gains access to the
communication medium through which some communication is taking place and then
listens to the communication and gets information about the content of the message. In
eavesdropping both the sender and the recipient can hardly notice that the data has been
stolen, intercepted or defaced.
25.
Hardware Software

Software is a program or a set of


Hardware is a et of physical parts of the
instructions which are to be executed by
computer
the computer.
Input devices, output devices, storage Programming software, application
devices, internal components of CPU , software, operating system are examples
motherboard are examples of hardware. of software.
It can be developed in machine language
It under stands only machine language
or assembly language or high level
or binary language
language
It remains unaffected by viruses Software are affected by viruses

Section – C
26. $0s^E $0s@#$0s$0s#$0s@@
27.
Absorption Law :- A . (A + B) = A (First Law)
A + (A . B) = A (Second Law)
A B A+B A . (A+B) A.B A + (A . B)
0 0 0 0 0 0
0 1 1 0 0 0
1 0 1 1 0 1
1 1 1 1 1 1
First Law Second Law
28.
l =[]
n = int(input("enter no. of students :- "))
for i in range(1, n+1) :
name = input("enter name of the student :-")
l.append(name)
t = tuple(l)
nm = input("enter name of the student which has to be searched:-")
if nm in t :
print("the name exist")
else :
print("the name does not exist")
10
OR
x = "y"
students = {}
while x == "y":
a = input("enter name")
b = input("enter roll number")
c = int(input("Enter marks"))
x = input("Enter 'y' to enter more values")
students[a] = (c,b)
print(“names of students getting more than 60”)
for z in students:
if students[z][0] >60:
print("Name is,", z)
29.
i) This act of the ill – intentioned person is called Stalking.
ii) The software designed by the person is a Spyware.
iii) The hardware device or software program is called Firewall.
30.
n=5
for i in range(n):
for j in range(i):
print('* ',end=' ')
print(" ")
for i in range(n, 0, -1):
for j in range(i):
print('* ',end=' ')
print(" ")
Section – D
31.
n = int(input("Enter number of calls made: "))
if n<=50:
print("No cost.")
elif (n>50) and (n<=200):
print("The total cost in rupees is",1.06*(n-50))
elif (n>200) and (n<=400):
print("The total cost in rupees is",1.06*(150 + (n-200)*4))
elif (n>400) and (n<=500):
print("The total cost in rupees is ",1.06*(150 + 200*4 + (n-400)*6))
else:
print("The total cost in rupees is",1.06*(150 + 200*4 + 100*6 + (n-500)*8))

11
32.
sum=0
f=1
s=1
n = int(input("enter number of terms"))
for i in range(2,n+2) :
f=f*i
s=s+i
sum=sum+(s/f)
print(sum)
Section – E
33.
a) Firewall.
b) Antivirus and anti – spyware solution.
c) Cookies.
d) Anti – spam software.
e) Authorization / Authentication / Firewall.
34.
a)
i) print(x[-7:-1]) = in Go
ii) print(x[::2]) = Hv at nGd
iii) print(x.index('in')) = 11
iv) print(x.replace("God","Supreme Power")) = Have faith in Supreme Power
b)
x = input("enter the line ")
s = 'is'
p = x.split()
c=0
for i in p:
if i == s:
c += 1
print(c)
OR
a)
i) print(x[-2::]) = od
ii) print(x[::-2]) = dGn ta vH
iii) print(x[-1:-7]) =
iv) print(x.split('i')) = ['Have fa', 'th ', 'n God']
v)

12
b)
x = input("enter the string ")
x = x.lower()
l = len(x)
print("original string in lower case : - ", x)
s =' '
for i in range(l):
if i%2==0:
s+=x[i]
else:
s+=x[i].upper()
print("the string after capitalizing every other letter :- ", s)
35.
a) The count ( ) of lists returns the count of the item that is passed as an argument to
the function. If the given item is not in the list it returns zero.
Syntax : <list name> . count(<item>)
b)
x = [10,'FUN',4,'HAPPY',90,'SAD',37,'ENJOY']
for i in range(len(x)):
if type(x[i])== int:
x[i] = x[i]**2
elif type(x[i]) == str:
x[i] = x[i].lower()
print(x)
OR
a) The fromkeys( ) is used to create a dictionary from a sequence containing all the
keys and a common value which would be assigned to all the keys.
Syntax : <dictionary name> = dict . fromkeys(<sequence containing key values>,
<common value>)
b)
x = [1,2,3,4,5,6,7,8,9,10]
print("original list :- ", x)
for i in range(len(x)):
if x[i]%2==0:
x[i]+=10
else:
x[i]+=5
print("list after modification :- ", x)



13
MODEL QUESTION PAPER
FINAL EXAMINATION
SESSION : 2023-24
SUBJECT : PHYSICAL EDUCATION
CLASS – XI

Time Allotted : 3 Hours Maximum Marks : 70


General Instructions:
 The question paper consists of 5 sections and 37 Questions.
 Section A – consists of question 1-18 carrying 1 mark each and is multiple
choice questions. All questions are compulsory.
 Section B – consist of questions 19-24 carrying 2 marks each and are very
short answer types and should not exceed 60-90 words. Attempt any 5.
 Section C – consist of Question 25-30 carrying 3 marks each and are short
answer types and should not exceed 100-150 words. Attempt any 5.
 Section D – consist of Question 31-33 carrying 4 marks each and are case
studies. There is no choice available.
 Section E – consists of Question 34-37 carrying 5 marks each and are long
answer types and should not exceed 200-300 words. Attempt any 3.
_____________________________________________________________________________________
SECTION - A
MCQ 1 – 18 (refer to the textbook questions)

SECTION - B
19. Explain social development as an objective of Physical Education. (1×2=2)
20. What does the Olympic motto – Citius, Altius, Fortius - mean? (2)
21. What is physical disability? (2)
22. Why is yoga important? (2)
23. What do you mean by coordinative abilities? (2)
24. Define the term Evaluation. (2)

1
SECTION - C
25. State any three functions of bones. (1×3=3)
26. What do you mean by Kapalbhati? (3)
27. Discuss the role of physiotherapist for children with special needs. (3)
28. Define term Cohesion and how it is useful. (3)
29. How narcotics effect the athletes? (3)
30. What do you mean by doping? Which two agencies regulate doping related activities?
(3)
SECTION - D
31. To 33. Case Studies paragraph questions (refer to textbook for examples)

SECTION - E
34. What are the guidelines for test administration? (1×5=5)
35. Write about the principles of Biomechanics. (1×5=5)
36. Explain the importance of sports psychology. (1×5=5)
37. Explain the terms Overload, Adaptation and Recovery in sports training. (5)

ANSWERS

SECTION - A

MCQ 1 to 18 (refer to the textbook as examples questions)

SECTION - B
19. Social development is an objective of physical education because:
It is related to the development of social traits essential for better adjustment in life.
It is one of the sources to attain leadership qualities like fair play, team spirit,
cooperation, tolerance, sportsmanship, and courtesy.
20. The Olympic motto - Citius, Altius, Fortius means – Faster, Higher, Stronger. These
three words encourage the athlete to give his or her best during competition.
21. Dysfunction of different body parts e.g., Mobility, dexterity or stamina become limited.
22. Good health is the right of every human being. But this right depends on individual’s
social and environmental factors. Health is positive concept. Positive health does not
mean merely freedom from disease, but it also includes an energetic feeling of well-
being with an amount of general resistance and capacity to easily cultivate immunity
against specific disease. Yoga is one of the most powerful systems of treatment free

2
from any form of medication or drugs. Yoga can be adopted as lifestyle for promoting
our physical and mental health.
23. The term coordinative ability is taken from the term agility. The term agility is discarded
because it was difficult to explain the meaning of it. It is the ability of the body to
perform movements with perfect efficiency. There are many kinds of abilities involving
coordination. It is a combination of strength, speed, endurance, visualisation, response,
balance and reflexes while in motion. It is neuro-muscular coordination and depends
upon the central nervous system.
24. Evaluation is a technique used to know the extent to which objectives are being
achieved. It is a decision-making process which assist to make grade and ranking.
According to Barrow and Mc Gee, “It is the process of education that involves collection
of data from the products which can be used for comparison with preconceived criteria
to make judgment.”
SECTION - C
25. Our bones perform seven important functions.
Support: Bones provide a framework that supports the body and cradles its soft organs.
For example, bones of the lower limbs act as pillars to support the trunk of the body
when we stand, and the rib cage supports the thoracic wall.
Protection: The fused bones of the skull protect the brain; the vertebrae surround the
spinal cord and the rib cage protect the vital organs of the thorax.
Movement: Skeletal muscles, which attach to bones by tendons, use bones as levers to
move the body and its parts. As a result, we can walk, grasp objects and breath. The
design of joints determines the types of movements possible.
26. The word kapalbhati is made up of two words,’kapal’ meaning skull (here skull includes
all the organs under the skull) and ‘bhati’ means shining or illuminating. Due to the
process, the organs under the skull mainly the brain and the small brain are influenced in
a good manner.
27. Physiotherapists are trained to evaluate and improve movement and function of the
body, with particular attention to physical mobility, balance, posture, fatigue, and pain.
The physical therapy programme typically involves educating the child about the
physical problems caused by their special needs, designing an individualised exercise
programme to address the problems, and enhancing mobility and energy conservation
using a variety of mobility aids and adaptive equipment.
28. According to Caren: Term cohesion is a “Dynamic process which reflected in the
tendency for group to stick together and remain united in the pursuit of goals and
objectives”. Team cohesion is useful as it can both improve performance and motivation
of a team. If a team work well together, this improves their team cohesion, this in turn
improves performance. This, then improves personal satisfaction. Thus, improving team
cohesion and then the cycle continues either on an upward or downward trend.

3
29. Side effects of consuming narcotics are:
Addiction
Loss of balance and coordination
Nausea dizziness
Insomnia and depression
Decreased heart rate.
30. “Doping is defined as the occurrence of one or more of the anti-doping rule violations
set forth in articles of code”.
The two agencies are WADA (World Anti – Doping Agency) and NADA (National
Anti-Doping Authority)

SECTION – D
31. To 33. Case Studies paragraph questions (refer to textbook as examples)
SECTION - E
34. Test administration guidelines in physical education and sports:
Test should be performed in a safe manner and the athletes should be monitored before,
during and after strenuous testing.
Information should be gathered about the testing session itself. The weather, time of the
day and specific testing setup should be recorded.
Test battery – a set of physical tests designed not to interfere with each other. Example:
Two non-fatiguing tests taken one right after another like height and flexibility.
To prepare athletes, they should be made aware of the date, time and purpose of the
testing process.
General and specific warm-up performed before a test can increase test reliability.
35. Principle of Stability: The lower the centre of mass, the larger the base of support, the
closer the centre of mass to the base of support and the greater the mass, the more
stability increases. Example: Sumo wrestling.
Principle of Maximum Effort: The production of maximum force requires the use of
all possible joint movements that contribute the task’s objective. Example, golf, bench
press
Principle of Maximum Velocity: The production of maximum velocity requires the use
of joints, in order – from the largest to smallest. Examples, hockey slapshot, hitting a
golf ball. Principle of Liner Motion: Movement usually occurs in the direction opposite
to that of the applied force. Examples, high jumper, cyclists, and runners.

4
Principle of Angular Motion: Angular motion is produced by application of a force
acting at some distance from an axis, that is torque. This principle is known as the
principle of the production of angular motion, example- baseball pitchers.
36. The performance of sportsperson not only depends on physiological factors, but also on
psychological factors. It helps to improve the sports performance. The psychological
process of motivation guidance helps in boosting the morale of athlete. This is the reason
why psychological training has become important in modern training programme.
Improving concentration: If a sportsperson loses self-confidence, if affects his
performance. Common distractions are negative thoughts, anxiety, mistakes, weather,
public annulments, fatigue, opponents, coach, etc. Strategies to improve concentration
are applied from sports psychology which helps the players in drawing more
concentration which leads to better performance.
Improving Self-Confidence: Sports psychology helps sportsperson to improve the self-
confidence to perform their maximum in sports. Confidence result from the comparison,
a sportsperson makes between a goal and his ability. When a sportsperson has self-
confidence, he will tend to maintain even when things are not going positive.
Better training methods: Every sportsperson has his own kind of attitude, interest, etc.
Thus, using sports psychology, an individual programme for each sportsperson
according to his need can be designed so that better result can be achieved.
Improvement in coaching skills: Sports psychology helps coaches to understand the
behaviour of sportsperson. Thus, he can train them in better way.
Motivation and feedback: Proper motivation and proper feedback enhances the
performance of players. It gives counselling to players. This can be guided by sports
psychology.
37. Overload: During training of sportsperson, load is given to the players according to their
capacity. Whenever this load goes beyond the capacity of an individual, the
physiological and psychological functions get disturbed. Through, this increased load
does not affect the sportsperson immediately, if the administration of the load continues
for a longer period, it results in decrease of his/her performance. The important signs and
symptoms of overload are:
Fatigue, Decline in performance., Loss concentration. Lack of motivation.
Adaptation: Adaptation is defined as the adjustment of physical and psychological
functional systems to the training load. Adaptation to a load result in the enhancement of
performance capacity.
Thus, a sports person can increase his/her performance as a result of adaption process.
Adaptation process demands that a sports person maintains regularity in training. If a
sports person is exposed to new and unfamiliar load in a systematic planned way the
adaption process will be faster.

5
Recovery: Recovery is to regain energy after workout what was lost during the activity.
A training plan must be designed in such a manner that proper rest and recovery can
take place in between the training sessions.



6
MODEL QUESTION PAPER
FINAL EXAMINATION
SESSION : 2023-24
SUBJECT : APPLIED ART
CLASS - XI
Time Allotted : 2 Hours Maximum Marks: 30
General Instructions:
Section-A
 Attempt all Questions (Each Question will carry 1 mark)
Section-B
 Attempt all Questions (Each Question will carry 2 Marks)
Section-C
 Attempt any two Questions (Each Question will carry 6 Marks)

SECTION – A
1.i) The first mosque in India is
a) Jama Masjid c) Tomb of Humayun
b) Taj mahal d) Qwawat-Ul-Islam
ii) Wizard dance was found from cave no.
a) 49 c) 47
b) 41 d) 43
iii) Bearded priest is made of
a) Steatite c) Red sandstone
b) Granite d) Sandstone
iv) Gandhara sculptural tradition had the confluence of the traditions of
a) Iran and Afghanistan c) China & Japan
b) Parthia, Bactria & local Gandhara d) Italy & Indian
v) The biggest rock cut chaitya hall was excavated from
a) Karla c) Nasik
b) Bhaja d) Junnar

1
vi) Sanchi temple has a
a) Vaulted roof b) flat roof
b) Hut shaped roof d) nagara type roof
vii) The inner chamber of Taj Mahal is in
a) Octagonal shape c) rectangular shape
b) Hexagonal shape d) spherical shape
viii) Harappa was discovered by
a) Rakhal Das Banerjee c) Alexander cumminghum
b) Dayaram Sahni d) Damien Hirst
SECTION – B
(Short answer type questions)
Answer for this question is expected in around 100 words
2. The lion capital of Ashoka is the national emblem of the republic of India. Justify the
reasons behind it.
3. Give an example of free standing sculpture of Mauryan Empire. Write your view about
it’s technical importance.
4. Discuss any painting of Ajanta cave.
5. What is pietra dura?
6. What is the earliest example of Cire perdue in India ? Why is it so important ? (2)
SECTION – C
Attempt any two questions from the given options
(Long answer type questions)
Answer for this question is expected in around 200 words
7. Differentiate Nagara and Dravida type temple. (6)
8. Why is Sanchi stupa considered as one the most important Buddhist architectures?
Explain. (6)
9. Give an example of a monolithic rock sculpture of Pallava dynasty and discuss its
compositional arrangement. (6)

2
ANSWER

SECTION - A
1. i) d) Qwawat-Ul-Islam
ii) c) 47
iii) a) steatite
iv) b) Parthia, Bactria and local Gandhara
v) a) Karla
vi) c) flat roof
vii) a) octagonal
viii) d) Dayaram Sahni

SECTION -B
2. The lion capital of Sarnath was built by Ashoka to preach Dhammachakkapavattana. It
has four parts, Chakra (which is kept separately), four lion sculptures appears aggresively
towards four directions, the abacas consists of four animals and wheels, fluted bell and
the shaft. The lions are considered as the king of animals, thus the Buddha was a lion
among all the spiritual masters. This magnificently executed work is an exceedingly
effective for Buddha’s cosmic preaching of the Law. The four animals, horse, bull, lion
and an elephant, curved on it’s abacus, depict four quarters of the world. It also carries
the evidence of the excellent skill of Mauryan polish. Undoubtedly it is a great piece of
Indian culture and accepted as the nation emblem of the republic India.
3. Didarganj yakshini is a great example of Mauryan empire. Yakshinis were considered as
holy spirits and used to be kept in front of door to welcome the worshipers. This life size
sculpture has heavy breast and hip .The curving is voluptuously realistic and each turn
or fold of indicated flesh has a slightly inflated sensuousness which paradoxically invests
the heavy stone with lightness.
It is made of sandstone. Mauryan sculptors were very skilled and they have enhanced it’s
beauty by adding the Mauryan gloss to it which gave an extra smoothness and polished
shiny surface. The figure doesn’t have any support or wall.
4. Padmapani bodhisatwa is one the remarkable creations of Ajanta sites. The central figure
is the image of Buddha who is shown in tri-bend or tribhanga pose. He is wearing a
crown and ornaments which clearly indicate that he belongs to a royal family or high
caste. He is holding a blue lotus in his right hand. There is aremote calmness in his face
which is enhanced by the figures in the background. These figures crowd in form of all
directions which establish bodhisatwa as an island of spiritual disengagement. The light

3
present in the painting is very subdued and divine. Absence of shadows also creates such
heavenly atmosphere.
5. Pietra Dura In the Indian Subcontinent, is a term for the inlay technique of using cut and
fitted, highly polished colored stones to create images. It is considered a decorative art.
The stonework, after the work is assembled loosely, is glued stone-by-stone to a
substrate after having previously been "sliced and cut in different shape sections; and
then assembled together so precisely that the contact between each section was
practically invisible". In the site of Taj Mahal some great works of Pietra Dura are found.
6. Dancing girl is the one of the earliest examples of Cire Perdue found in Mohenjodaro.
Inspite of its small size this jwell of realism is completely urban in pose and hauteur. It’s
right hand is in her right hip and left hand holds a bowl. The statue is in tribhanga pose
and left leg is slightly bended. All these features suggest that it is a figure of a dancing
girl. The hair is tied in a knot and the figure has few ornaments on neck and hands.
Dancing girl is an evidence of the use of Cire Perdue in early days and it gives us
valuable informations about that period.
SECTION-C
7. Differentiate Nagara and dravida temple style.
The northern temple style and southern temple style are discarded in favour of
indigenous level, Nagara and Dravida respectively. The distinction rests on the main
features : the tower, Surmounting the sanctum, the ground plan and the elevation or
external walls.
 The Nagara tower (shikhara ) has a gently sloping curve, with a fluted disc (amalaka)
at the pinnacle. The Dravida tower (Vimana) follows a dome and cornice pattern like
a pyramid with diminishing tiers (tala), crowned by a square, polygonal or round
dome.
 The Nagara elevation consists of a series of projections (ratha) and recesses whereas
the walls of Dravida temples are superficially similar to European buildings in being
broken up by images within entablatures at regular intervals.
 Unlike the nagara temple, the dravida temple is enclosed within a compound wall.
The front wall has an entrance gateway in its centre, which is known as a gopuram.
 Whereas at the entrance to the North Indian temple’s garbhagriha, it would be usual to
find images such as mithunas and the river goddesses, Ganga and Yamuna, in the
south sculptures of fierce dvarapalas or the door-keepers guarding the temple are
found.
8. Sanchi, about 50 km from Bhopal, the capital of Madhya Pradesh,is a world heritage
site. It is undoubtedly one of the best Buddhist architectural creations. Along with
other relatively small stupas, in the contours gets reduced and images are
givenmovement.

4
 Carving techniques appear more advanced than Bharhut. Symbols continue to be used
representing the Buddha.
 The historical narratives such as the siege of Kushinara, in the contours gets reduced
and images are given movement. Narration gets elaborated; however, the depiction of
the dream episode remains very simple showing the reclining image of the queen and
the elephant at the top.
 Originally the stupa was a small brick structure which expanded over a period and
was covered with stone, vedika and the torana (gateways). The Ashokan lion- capital
pillar with an inscription is found on the southern side of the stupa, indicating how
Sanchi became a centre of monastic and artistic activities.
 The south gateway was made first followed by the others. The pradakshinapath
around the stupa is covered with the vedika. There is also the upper pradakshinapath
which is unique to this site. The four gateways are decorated profusely with
sculptures. Buddha is shown symbolically as an empty throne, feet, chhatra, stupas,
etc. Toranas are constructed in all four directions.
 The figures of yaksha and yakshinisare more three dimensional and seem to step from
the background.
9. Descent of the Ganges is a monument at Mamallapuram, on the Coromandel Coast of the
Bay of Bengal, in the Chengalpattu district of the state of Tamil Nadu, India. Measuring
96 by 43 feet, it is a giant open-air rock relief carved on two monolithic rock boulders.
The legend depicted in the relief is the story of the descent of the sacred river Ganges to
earth from the heavens led by Bhagiratha. The waters of the Ganges are believed to
possess supernatural powers. The descent of the Ganges and Arjuna's Penance are
portrayed in stone at the Pallava heritage site. The relief is more of a canvas of Indian
rock cut sculpture at its best not seen anywhere in India. It is one of the Group of
Monuments at Mamallapuram that were designated as a UNESCO World Heritage Site
since 1984.
It is believed to be the depiction of Arjuna, performing penance to receive the Pasupatha
Astra (the god-slayer) to defeat the Kauravas. Shiva, pleased with his penance is seen
granting him this wish.
The same ascetic, is believed by some historians to be Sage Bhagirath (ancestor of Ram)
performing penance to seek Shiva’s help in bringing the River Ganga from the heavens to
earth. Legend has it, that Bhagirath’s ancestors of the Ikshavu Dynasty were turned into
ashes following the curse of sage Kapila (they had disturbed his meditation). The sage
also gives an antidote to his curse – if the ashes of ancestors were immersed in the Ganga,
their souls would attain Moksha. Bhagiratha’s penance has Shiva agreeing to bring Ganga
down to earth through his hair-locks so as to tone down her vigour and size.



5
MODEL QUESTION PAPER
FINAL EXAMINATION
SESSION : 2023-24
SUBJECT : CORE MATHEMATICS
CLASS - XI
Time Allotted : 3 Hours Maximum Marks : 80

General Instructions:
 The question paper contains five sections A, B, C, D and E. Each section is
compulsory.
 Section – A has 20 questions of 1 mark each.
 Section – B has 5 questions of 2 marks each.
 Section – C has 6 questions of 3 marks each.
 Section – D has 4 questions of 5 marks each.
 Section – E has 3 case based questions of 4 marks each.

SECTION – A
(All questions are compulsory) (1 × 20 = 20)
3 9  
1. If sin  A   ;cos B= ;0<A< ;0<B< value of sin  A–B  is
5 41 4 4
133 133
a) b)
205 205
132 132
c) d)
205 205
2. The solution set of x for the inequation :
x –3
 0 is
x–5
a) x  (–, 3) b) x  (5, )
c) x  (3, 5) d) x  (– ,3)   5,  
1 1 x
3. If   the value of x.
9! 10! 11!
a) 100 b) 120
c) 121 d) 125

1
4. The equation of Circle whose centre is (2, –3) and radius = 8 units is
a) x 2  y 2  4 x  6 y  51  0 b) x 2  y 2  4 x  6 y  51  0
c) x 2  y 2  4 x  6 y  51  0 d) x 2  y 2  4 x  6 y  51  0
5. The equation of directrix for y2 = 8x is
a) x = 8 b) x = –8
c) x = –2 d) x = 2
6. The length of smallest focal chord of 16x + 25y2 = 400 is
2

28 28
a) b)
5 3
26 32
c) d)
5 5
2 2
7. The length of transverse axis of 16x – 9y = 144 is
a) 6 b) 4
c) 2 d) None of these
8. The distance between P(–2, 4, 1) and Q(1, 2, –5) is
a) 7 units b) 9 units
c) 10 units d) 12 units
dy
9. If y = x3 – 27, then is
dx
a) 3x3 b) 3x2
c) 3x2 – 27 d) x3
10. The probability of getting two red cards from pack of cards is
13 4
C C2
a) 52 2 b) 52
C2 C2
26
C2
c) 52
d) None of these
C2

11. Find the domain of f(x) = 25  x2


a) (–5, 5) b) [–5, 5]
c) [5,) d) (–,)
12. The co-ordinates of vertex of y 2 – 8 y – x  19  0 is
a) (4, 3) b) (–4, –3)
c) (–3, –4) d) (3, 4)
13. The eccentricity of ellipse 5 x 2  9 y 2  1 is
2 1
a) b)
3 2
2 3
c) d)
5 5

2
14. Polar form of (1 + i) is
a) 2  cos 45  i sin 45  b) 2  cos 45  i sin 45 
c) 2  cos 30  i sin 30  d)  cos 45  i sin 45 
15. The equation of circle if centre (2, 3) and radius = 2 units is
2 2 2 2
a)  x  2    y  3  4 b)  x  3   y  2   4
2 2 2 2
c)  x  2    y  3  4 d)  x  2    y  3  9

x2 y2
16. If   1  0 represents on ellipse, then
2–  –5
a)   [2, 5] b)   [2, 5)
c)   (2, 5) d)   (2, 5]
17. Given t1 = 1; tn = 3tn–1 + 2 for n > 1, then t3 is
a) 22 b) 24
c) 26 d) 28
18. Let A = {1, 2, {3, 4}, 5}. Which of the following is correct?
a) {3, 4}  A b) {1, 2, 5}  A
c) {3, 4}  A d) {}  A
ASSERTION-REASON QUESTIONS
Choose the correct answer out of the following choices:
a) Both A and R are true & R is the correct explanation of A.
b) Both A and R are true & R is not the correct explanation of A.
c) A is true & R is false.
d) A is false & R is true.
19. Assertion (A) : If n(A) = 4 & n(B) = 3 then n (A × B) = 4
Reason (R) : n(A × B) = n(A). n(B)
20. Assertion (A) : f(x) = x2, f is a function on Natural numbers.
Reason (R) : All relations are functions.

SECTION – B
(Each Question carry 2 marks) (2 × 5 = 10)
       
21. Prove that cos  – A  cos  – B  – sin  – A  sin  – B   sin  A+B 
4  4  4  4 

1
22. Solve: tan  
3
23. Find equation of circle which passes through touches y -axis at a distance 4 units from
origin and cuts an intercept of 5 units along the positive direction of x –axis.
dy x  ex
24. Find if y 
dx 1  ln x

3
25. A bag contains 9 red, 7 white and 4 black balls. If two balls are drawn at random find
the probability that both the balls are red?

SECTION – C
(3 × 6 = 18)
26. Find Equation of the parabola whose focus is (–3, 2) and directrix is x  y  4 ?
25
 1 
27. Find 11th term from end in expansion  2x  2 
 x 
28. Find co-ordinates of the point-R which divides the join of P(2, –1, 4) and Q(4, 3, 2) in
ratio 2 : 3 internally.
29. Find graphically the solution set of linear In Equations 3 x  4 y  12 , 4 x  3 y  12 ,
x0,y0
30. In how many ways can the letters of word PENCIL be arranged so that
i) N is always next E
ii) N and E are always together?
31. How many different words can be formed with letters of word MISSISSIPPI? In how of
these permutation four I’s do not come together?
SECTION – D (5 × 4 = 20)
32. A committee of 12 is to be formed from 9 women and 8 men. In how many ways this
can be done if at least five women to be included in a committee? In how of these
committee women are in majority? In how of many of these men are in majority?
33. Determine the point in xy plane which is equidistant from three points
A(2, 0, 3), B(0, 3, 2) and C(0, 0, 1)
ex 
34. i) Find the derivative of w.r. to x at x 
1  sin x 2
i 1
ii) Find modulus of
 
cos  i sin
3 3
–3
35. If cos  –    cos   –    cos   –    Prove that
2
cos   cos   cos   sin   sin   sin   0
SECTION – E (4 × 3 = 12)
36. Case Study - 1
Seema observed the sum of cardinal number of two finite sets A and B is 9. Kavya
observed ratio of cardinal number of power set A is to cardinal number of power set of
B is 8 : 1.
i) Find the cardinal number of A.
ii) Find the cardinal number of B.
iii) Find maximum value of n  A  B

OR
Find minimum value of n  A  B

4
37. Case Study - 2
A 5 digit number is formed. One number is choosen at random.
Based on above information answer the following questions.
i) Find the probability of choosing an odd number.
ii) Find the probability of choosing a number divisible by 5.
iii) Find the probability of choosing a number whose sum of digits is odd.
38. Case Study - 3
An auditorium has 20 seats in the first row, 24 sets in the school raw and 28 seats in the
3rd row and so on.
Based on above information, answer the following questions :
i) Find how many seats are there in the 16th row.
ii) In the last row of the auditorium there are 116 seats. Find how many rows are there
in the auditorium.
iii) Find the total number of seats in the auditorium.
OR
The hall was fill on Saturday for a show, find the total collection (in rupees) for the day,
if a ticket was sold for `200.

ANSWER

4 40
1.b) cos A = ;sin B =
5 41
133
sin  A–B   –
205
x –3
2.d) 0
x–5
+ +
3 5
x   – ,3    5,  
1 1 x
3.c)  
9! 10  9! 11 10  9!
x = 121
2 2
4.a)  x – 2   y  3   64

 x 2  y 2 – 4 x  6 y – 51  0

5
5.c) y 2  8 x
Equation of directrix is x = – ax = – 2
x2 y 2
6.d)  1
25 16
2b 2 2  16 32
Length of LR =  
a 5 5
7.a) 16 x 2 – 9 y 2  144 Length of Trans Verse Axis = 2a = 2× 3 = 6

x2 y 2
– 1
9 16
8.a) PQ= 9  4  36  7 units
9.b) y = x3 – 27
dy
 3x 2
dx
26
C
10.c) P(A) = 52 2
C2
11.b) [–5, 5]
2
12.d)  y – 4    x – 3 
Vertex (3, 4)
x2 y 2
13.a)  1
1 1
5 9

b2 1 4 2
e  1– 2  1– 5  
a 9 9 3

14.b) 2  cos 45  i sin 45 


2 2
15.c)  x – 2    y – 3  22
x2 y
16.c)  1
 –2 5–
 – 2  0 and 5 –   0
  2 and  <5
2<   5
   2,5 
17. c)
18. c)
19. d)

6
20. c)
  
21. cos  – A+ – B 
4 4 
 
= cos  –  A  B  
2 
=sin(A + B)
1
22. tan  
3

  n  ; n  Z
6
2 2
23.  x – 5   y  4  5
1
dy
1  ln x  1  e x  –  x  e x   
24.   x
dx 2
1  ln x 
x x ex
1  e  ln x e  1   ln x
x
2
1  ln x 
 1
e x 1  ln x    ln x
 x
1  ln x 2

25. 9 red
7 white
4 black
Total No. of balls = 9 + 7 + 4 = 20
9
C 18
P(2 red balls) = 20 2 
C2 95
26. SP2 =PM2

P(x, y)
M

S(–3, 2)

2
2 2 x y–4
  x  3   y – 2  
2

7
27. 11th term from end = (26 – 11 + 1)th = 16th term from beginning
15
2515  1 
T16  T151  25 C15  2 x   2
x 
1
=  25 C15  210 
x 20

25 210
=  C15  20
x
2  4  3  2 14
28. x 
23 5
2  3  3  –1 3
y= 
23 5
2  2  3  4 16
z= 
23 5
 14 3 16 
R=  , , 
5 5 5
x 0 4
29. 3 x  4 y  12
y 3 0

x 3 0
4 x  3 y  12
y 0 4

4
3

1
X' X
0 1 2 3 4

Y'

8
30. i) Taking EN as single letter there are 5 letters to be arranged in
5
P5  5!  120 ways

ii) No. of ways of arrangement = 5! × 2! = 240 ways


11!
31. Total No. of words =  34650
4!4!2!
No. of words in which
8!
41’s come together =  840
4!2!
 No. of words in which 41’s do not come together
= 34650 – 840 = 33810
32. No of ways the committee contains atleast 5 women are
9
C5 × 8C7 + 9C6 × 8C6 + 9C7 × 8C5 + 9C8 × 8C4 + 9C9 × 8C3
= 6062 ways
No of ways of forming the committee when women are in majority
9
C9 × 8C3 + 9C8 × 8C4 + 9C7 × 8C5
= 56 + 630 + 2016 = 2702 ways
No of ways of forming the committee when men are in majority
9
C5 × 8C7 = 1008 ways
33. Any point in xy plane P(x, y, 0)
 PA = PB and PB = PC
 x  2 2  y 2   0  3 2   x  0 2   y  3 2   0  2 2
 x 2  4 x  4  y 2  9  x2  y 2  6 y  9  4
 6 y  4x  0
 2 x  3 y  0 ……….(1)
Now PB = PC
2 2 2 2 2 2
  x  0    y  3   0  2    x  0    y  0    0  1

 x 2  y 2  6 y  9  4  x2  y 2  1
 6 y  12  y  2 ……………….(2)
Solving (1) and (2) P (3, 2, 0)
x
ex dy e 1  sin x  cos x 
34. i) y  
1  sin x dx 1  sin x 2

  
e2  1  sin  cos 
dy  2 2
 2
dx x

 
2 1  sin 2 
 

9
 

e 2 1  1  0  e 2  2 1 2
  e
1  12 4 2

i 1 i 1 2  1  i 
ii) Let Z   
  1 i 3 1 i 3
cos  i sin 
3 3 2 2

=

2  1  i   1  i 3   2  1  3   i 1  3 
2
12   i 1 3
3 
 3 1   3 1 
=    i  
 2   2 
2 2
 3 1   3  1  2  3  1
Z         2
 2   2  4
3
35. cos       cos      cos      
2
3
 cos  cos   sin   sin   cos  cos   sin  sin   cos  cos   sin  sin  
2

 2cos  cos   2sin   sin   2 cos  cos   2sin  sin   2cos  cos   2sin  sin   3

 2cos  cos   2sin   sin   2 cos  cos   2sin  sin   2 cos  cos   2sin  sin 
+ sin 2   cos 2   sin 2   cos 2   sin 2   cos 2   0
2 2
  cos   cos   cos     sin   sin   sin    0
cos   cos   cos   0
and
sin   sin   sin   0
36. m(A) + n(B) = 9……………(1)
2n(A) 8
n  B
  2n(A)n(B)  23  n(A)  n(B)  3 ………………(2)
2 1
From (1) & (2)
(i) n(A) = 6 (ii) n(B) = 3
(iii) n(AB) is maximum when n(AB) is minimum
 n(AB) = n(A) + n(B)  n  A  B   0 
=6+3
=9
OR
n(AB) is minimum when n(AB) is maximum
n(AB) = 3
 n(AB) = 6 + 3 – 3 = 6

10
37.i) There are 5 odd digits : 1, 3, 5, 7, 9
The number will be odd if it has odd digits at its unit place. So unit place can be filled
in 5 ways by an odd digit
Number of 5-digit odd numbers = 9 × 10 × 10 × 5 = 45000
No of 5  digit odd numbers
Required Probability =
Total no of 5  digit number
45000 1
= 
90000 2
ii) Number divisible by 5 should contain 0 or 5 at its unit place. Which can be filled in 2
ways.
Number of 5 digit no. divisible by 5 = 9 × 10 × 10 × 10 × 2 = 18000
18000 1
Reqd. Probability = 
9000 5
iii) If sum of digits at first four places is even, then fill the unit place by an odd digit, if the
sum of first four digit at first four places is odd, fill the unit place by an even digit. In
both these are 5 ways to fill the unit place.
45000 1
Required probability = 
90000 2
38.i) 20, 24, 28, ....... form an A.P. series
T16 = a + 15d ; a = 20
20 + 15 × 4 d = 4
= 80
ii) Tn = a + (n – 1)d Given Tn = 116
116 = 20 + (n – 1)4 a = 20
 96 = (n – 1)4 d=4
 24 = n – 1
 n = 25  There are 25 rows
iii) Total no of seats
25
S25 =  2  20   25  1 4 
2 
25 25
=  40  96  136
2 2
= 1700
Total no of seats = 1700
OR
Total Collections = 1700 × 200 = `3,40,000



11
MODEL QUESTION PAPER
FINAL EXAMINATION
SESSION : 2023-24
SUBJECT : PSYCHOLOGY
CLASS – XI
Time Allotted : 3 Hours Maximum Marks : 70
General Instructions:
 All questions are compulsory except where internal choice has been given.
 Question Nos. 1 -15 in Section A carry 1 mark each.
 Question Nos. 16-21 in Section B are Very Short Answer Type-I questions carrying
2 marks each. Answer to each question should not exceed 30 words.
 Question Nos. 22-24 in Section C are Short Answer Questions Type-II carrying 3 marks
each. Answer to each question should not exceed 60 words.
 Question Nos. 25-28 in Section D are Long Answer Type I questions carrying 4 marks
each. Answer to each question should not exceed 120 words.
 Question No. 29-30 in Section E is a Long Answer Type II question carrying 6 marks.
Answer to this question should not exceed 200 words.
 Question Nos. 31-36 in Section F are based on total 6 marks.

SECTION – A (1×15=15)
1. Tina is praised for her good behavior immediately after helping a classmate. What type
of learning principle is being applied in this scenario?
a) Classical conditioning
b) Operant conditioning
c) Observational learning
d) Insight learning
2. Jack perceives a distant mountain as smaller than it actually is. What perceptual concept
explains this phenomenon?
a) Size constancy
b) Shape constancy
c) Color constancy
d) Perceptual set

1
3. Dr. Roy conducts a study comparing memory recall in young and older adults. What
type of research design is Dr. Roy likely using?
a) Cross-sectional study
b) Longitudinal study
c) Case study
d) Experimental study
4. Jason can remember details from a traumatic event more vividly than everyday
experiences. What phenomenon is associated with this selective memory?
a) Flashbulb memory
b) Source amnesia
c) Retrograde amnesia
d) Anterograde amnesia
5. Mary applies psychological principles to optimize employee performance and well-being
in her workplace. What subfield of psychology does Mary likely specialize in?
a) Clinical psychology
b) Industrial-Organizational psychology
c) Health psychology
d) Forensic psychology
6. Tom observes his friend receiving praise for helping others, leading Tom to engage in
similar prosocial behavior. What type of learning is demonstrated in this scenario?
a) Classical conditioning
b) Operant conditioning
c) Observational learning
d) Latent learning
7. During a magic show, the magician uses visual illusions to create misleading
perceptions. What term best describes this intentional distortion of perception?
a) Perceptual constancy
b) Illusory motion
c) Hallucination
d) Visual illusion
8. Professor Davis conducts a study investigating the correlation between exercise and
mood in a sample of adults. What type of research method is Professor Davis using?
a) Experimental research
b) Correlational research
c) Longitudinal study
d) Case study

2
9. Jessica vividly recalls the details of her first day at a new job, including the people she
met and the conversations she had. What type of memory is Jessica utilizing in this
scenario?
a) Explicit memory
b) Implicit memory
c) Episodic memory
d) Semantic memory
10. Dr. Patel focuses on helping individuals cope with stress, anxiety, and other emotional
challenges. What subfield of psychology is Dr. Patel likely specialized in?
a) Clinical psychology
b) Counseling psychology
c) Social psychology
d) Educational psychology
11. According to Piaget’s cognitive development theory, what cognitive ability typically
emerges during the formal operational stage?
a) Object permanence
b) Abstract thinking
c) Conservation
d) Sensorimotor coordination
12. Robert experiences a surge of motivation to study for exams after setting a clear goal of
achieving a high GPA. What psychological concept is reflected in Robert's situation?
a) Intrinsic motivation
b) Extrinsic motivation
c) Drive reduction theory
d) Self-determination theory
13. Emily learns to associate the smell of freshly baked cookies with a feeling of warmth and
comfort. What type of learning is demonstrated in this classical conditioning scenario?
a) Habituation
b) Operant conditioning
c) Observational learning
d) Classical conditioning
14. Dr. Carter explores how social factors influence individual behavior in various settings.
What subfield of psychology is Dr. Carter likely specialized in?
a) Social psychology
b) Clinical psychology
c) Forensic psychology
d) Health psychology

3
15. Which type of thinking involves exploring various possibilities, being open to new ideas,
and often associated with the creative process?
a) Analytical thinking
b) Divergent thinking
c) Convergent thinking
d) Reflective thinking
SECTION – B (2×6=12)
16. Highlight the importance of EQ.
17. What are the major obstacles in problem solving?
18. Discuss the steps as how language is formed.
19. You are a learning specialist. Recommend memory enhancement techniques rooted in
cognitive psychology for college students preparing for exams. Provide specific
strategies, and explain how these techniques align with principles of memory storage and
retrieval.
20. You are a high school teacher aiming to improve students’ problem-solving skills. Apply
insights from Piaget’s theory of cognitive development to design a lesson plan on any
topic that encourages abstract thinking and hypothetical reasoning.
21. You are a childcare provider working with infants. Explain how understanding object
permanence is crucial for infants’ cognitive development. Provide examples of daily
activities and interactions that can support the development of object permanence in
infants under your care.
OR
Describe the concept of random sampling in research. Why is it important, and how does
it contribute to the generalizability of study findings?
SECTION – C (3×3=9)
22. In today’s digital age, discuss the potential impact of technology on cognitive
development in preschool-aged children. What considerations should parents and
educators keep in mind regarding screen time, and how can technology be used to
enhance cognitive skills during this developmental stage?
OR
Provide an example from everyday life that illustrates the phi phenomenon.
23. Analyze the potential impact of social media on the cognitive development of
adolescents. Discuss how exposure to diverse perspectives, information processing, and
the development of abstract thinking may be influenced by social media use. Suggest
guidelines for parents to facilitate healthy cognitive development in this digital age.
24. In the realm of advertising, analyze how marketers utilize perceptual principles to
influence consumer behavior. Provide examples of advertisements that leverage concepts
such as selective attention, figure-ground relationships, or perceptual grouping.
SECTION – D (4×4=16)
25. Discuss the advantages and disadvantages of using surveys as a research method. What
considerations should researchers keep in mind when designing surveys?

4
26. Emily, a high school student, is trying to choose her career path and is intrigued by the
field of psychology. She approaches a school counselor seeking guidance on what
psychology is and its diverse. As a school counselor, explain to Emily what psychology
is as a scientific discipline. Provide examples of the diverse topics and areas of study
within psychology.
27. Discuss the implications of spontaneous recovery for the effectiveness of extinction.
What are the factors that hinder extinction?
28. What are the methods of studying verbal learning?
OR
Suggest a mixed-methods approach that integrates both qualitative and quantitative data
collection methods to comprehensively evaluate the effects of the new teaching method.
SECTION – E (2×6=12)
29. Explore research-based techniques for improving memory. Discuss cognitive strategies,
mnemonic devices, or interventions that have been shown to enhance memory
performance. How can individuals apply these techniques in their everyday lives?
OR
Analyze the phenomenon of false memories and its implications for eyewitness
testimony. Discuss factors that contribute to the creation of false memories and explore
the challenges of relying on eyewitness accounts in legal contexts. What measures can
be taken to minimize the influence of false memories in legal proceedings?
30. What is the relation between decision making and judgment? State examples as how
judgment may influence decision making.
OR
In a team-based project, explain how a combination of analytical, creative, and critical
thinking can enhance the overall problem-solving process. Provide specific examples of
how each type of thinking contributes.
SECTION – F (6)
Sarah, a college student, often finds her mind wandering during lectures.
31. The concept of “mind wandering” is a part of _________ attention. (1)
32. How might mind wandering impact task performance? (2)
Ahmed, an international business executive, recently moved to a new country for work.
Describe the cultural factors that may influence how Ahmed expresses and interprets
emotions in his new environment.
33. Discuss the potential challenges and advantages he might face in adapting to different
cultural norms regarding emotional expression. (1)
34. Suggest strategies for Ahmed to navigate and integrate into the new cultural context
while effectively managing his emotional expressions. (2)

5
ANSWERS
SECTION – B
16.
 Facilitates effective interpersonal relationships
 Enhances success in personal and professional contexts
 Involves recognizing, understanding, and managing emotions
 Perceiving and influencing others' emotions
17.
 Mental set
 Functional fixedness
18.
 Babbling
 One-word stage
 Two-word stage
 Telegraphic speech
 Complex sentences
 Learning through imitation, reinforcement, and exposure
19.
 Active recall
 Spaced repetition
 Mnemonic devices
 Visualization
 Engages with information repeatedly
 Creates associations and taps into multiple senses
20.
 Challenging, open-ended problems
 Encouraging exploration
 Guiding discussions
 Fostering abstract thinking and hypothetical reasoning
 Promoting cognitive development for advanced problem-solving
21.
 Understanding that objects exist even when out of sight
 Activities like peek-a-boo reinforce this concept
 Consistent, responsive interactions create a secure environment
 Supports gradual development of object permanence
 Enhances overall cognitive growth
6
OR
 Random sampling is a research method where every member of the population has an
equal chance of being included in the study.
 Participants are selected randomly, without any bias or predetermined pattern.
SECTION – C
22.
 Limiting excessive screen exposure
 Emphasizing interactive and educational content
 Monitoring content appropriateness
OR
 Educational apps promoting literacy and numeracy
 Interactive games fostering problem-solving
 Balancing screen time with hands-on activities
 Parental guidance and involvement are crucial
Write with context to Piaget.
23. Social Media's Impact on Adolescent Cognitive Development:
Exposure to Diverse Perspectives:
 Encourages exposure to global viewpoints
 Can broaden understanding and tolerance
Information Processing:
 Rapid information dissemination
 Challenges critical evaluation skills
Development of Abstract Thinking:
 May expose adolescents to complex ideas
 Requires guidance to navigate and comprehend
Guidelines for Parents:
 Open communication about online experiences
 Encouraging critical thinking and fact-checking
 Setting limits on screen time
 Monitoring social media interactions
24. Perceptual Principles in Advertising:
Selective Attention:
 Use of attention-grabbing elements
 Highlighting key product features
Figure-Ground Relationships:

7
 Creating visual contrasts to emphasize a product
 Making the product stand out against the background
Perceptual Grouping:
 Arranging elements to form cohesive visual units
 Enhancing overall visual appeal
Examples:
 Coca-Cola's red against a neutral background
 Apple's minimalist design drawing attention to products
Impact on Consumer Behavior:
 Influencing perceptions and preferences
 Shaping consumer attitudes and purchase decisions
SECTION – D
25. Advantages and Disadvantages of Surveys:
Advantages:
 Efficiency: Can collect data from a large sample quickly.
 Standardization: Allows for uniform data collection.
 Anonymity: Participants may feel more comfortable sharing sensitive information.
Disadvantages:
 Response Bias: Participants may provide socially desirable answers.
 Limited Depth: May not capture complex emotions or experiences.
 Question Wording: Ambiguous or leading questions can skew results.
Considerations for Survey Design:
 Clear and Unbiased Questions: To avoid misinterpretation.
 Random Sampling: To ensure representative data.
 Pilot Testing: Identifying and addressing potential issues.
 Ethical Considerations: Protecting participants' privacy and well-being.
26. Introduction to Psychology as a Scientific Discipline:
 Definition: Psychology is the scientific study of behavior and mental processes.
Diverse Topics and Areas:
 Clinical Psychology: Focuses on mental health and therapy.
 Cognitive Psychology: Examines mental processes like memory and problem-
solving.
 Social Psychology: Studies how individuals interact in social settings.
 Developmental Psychology: Explores human growth and changes across the
lifespan.

8
27. Implications of Spontaneous Recovery in Extinction
 Definition: Spontaneous recovery is the reappearance of an extinguished response.
Implications:
 Effectiveness of Extinction: Can be temporary, impacting long-term success.
Factors Hinder Extinction:
 Contextual Cues: Environmental factors triggering recovery.
 Timing: Delay between extinction and assessment.
28. Methods of Studying Verbal Learning:
 Paired-Associate Learning: Associating pairs of words.
 Serial Learning: Memorizing items in a specific order.
 Free Recall: Recalling information without specific cues.
 Repetition and Rehearsal: Emphasizing the role of repetition in verbal learning.
OR
Mixed-Methods Approach for Evaluating Teaching Method:
Quantitative Methods:
 Surveys/Questionnaires: To gather numerical data on student perceptions.
 Test Scores: Assessing academic performance.
Qualitative Methods:
 Interviews/Focus Groups: Exploring in-depth insights on teaching experiences.
 Observations: Understanding classroom dynamics and engagement.
 Integration: Combining both types of data for a comprehensive understanding of the
teaching method's effects.
SECTION – E
29. Techniques for Improving Memory:
Cognitive Strategies:
 Chunking: Grouping information into manageable chunks.
 Visualization: Creating mental images to aid recall.
 Association: Linking new information to existing knowledge.
 Mnemonic Devices:

 Acronyms: Creating memorable abbreviations.


 Rhymes and Songs: Using musical elements for recall.
 Method of Loci: Associating information with specific locations.
Interventions:
 Mindfulness Practices: Enhancing focus and attention.
 Regular Exercise: Promoting overall brain health.
9
 Adequate Sleep: Consolidating and storing memories.
Application in Everyday Lives:
 Study Habits: Implementing cognitive strategies during learning.
 Daily Routines: Utilizing mnemonic devices for remembering tasks.
 Healthy Lifestyle Choices: Incorporating interventions for overall cognitive well-
being.
OR
Phenomenon of False Memories and Eyewitness Testimony:
Definition of False Memories:
 Inaccurate recollections of events that never occurred.
Implications for Eyewitness Testimony:
 Eyewitnesses may confidently testify about events that did not happen.
 Legal cases can be significantly impacted by the credibility of eyewitness accounts.
Factors Contributing to False Memories:
 Leading questions or suggestive language during questioning.
 Exposure to misleading information that distorts memory.
 Difficulty distinguishing between actual and suggested events.
Challenges in Legal Contexts:
Eyewitness Confidence:
 Confidence in false memories can influence juror perceptions.
Impact on Legal Outcomes:
 Innocent individuals may be wrongly convicted based on inaccurate testimony.
Measures to Minimize Influence in Legal Proceedings:
Cognitive Interviews:
 Use of non-leading, open-ended questions during investigation.
 Informing the court about the fallibility of human memory.
 Providing information on the factors influencing memory reliability.
Conclusion: False memories pose significant challenges to the reliability of eyewitness
testimony in legal proceedings. Implementing careful investigative techniques, lineup
procedures, expert testimony, and juror education can help minimize the impact of false
memories, ensuring a more accurate and just legal system.
30. Relationship Between Decision Making and Judgment:
 Judgment Definition: Evaluation or interpretation of information.
 Decision Making Definition: Choosing a course of action.
Examples:
Tendency to favor information confirming existing beliefs.
10
Influence on Decision: May lead to biased choices based on preconceptions.
Overestimating one's abilities or the accuracy of judgments.
Influence on Decision: Can lead to risky choices due to unwarranted confidence.
Relying too heavily on the first piece of information encountered.
Influence on Decision: Sets a reference point that can bias subsequent choices.
OR
Team-Based Project and Thinking Styles:
Analytical Thinking:
 Example: Analyzing data to identify trends and patterns.
Creative Thinking:
 Example: Brainstorming innovative solutions to a problem.
Critical Thinking:
 Example: Evaluating the strengths and weaknesses of different approaches.
Integration:
 Example: Using analytical skills to assess data, creative thinking to generate ideas,
and critical thinking to select the most viable solution.
Enhanced Problem-Solving:
 Example: A team incorporating diverse thinking styles can develop comprehensive
and effective solutions to complex problems.
SECTION – E
31. Concept of "Mind Wandering" and Attention:
 Answer: The concept of "mind wandering" is a part of sustained attention.
32. Impact of Mind Wandering on Task Performance:
 Mind wandering can significantly impact task performance as it leads to a decrease in
attention and engagement with the task at hand. It may result in reduced
comprehension, learning, and overall effectiveness in completing tasks.
33. Cultural Factors Influencing Emotional Expression:
 The cultural factors that may influence how Ahmed expresses and interprets emotions
include cultural norms, communication styles, and the degree of emotional
expressiveness accepted in the new environment.
34. Adapting to Different Cultural Norms:
Challenges:
 Recognizing and understanding cultural nuances related to emotional expression.
 Navigating potential misunderstandings due to different expectations.
Advantages:
 Building stronger interpersonal connections by aligning with local norms.
 Enhancing cross-cultural communication and collaboration.

11
Strategies for Integration:
 Observation and Adaptation:
 Observe local practices and adjust expressions accordingly.
Cultural Sensitivity Training:
 Engage in training to understand and respect cultural differences.
Seeking Guidance:
 Consult with local colleagues or mentors for insights.
Effective Communication:
 Clearly communicate emotions to bridge cultural gaps while staying true to personal
values.
NOTE: For 4 and 6 marks questions, please elaborate on the points outlined. Use proper key
words, sub heads in writing the answers. Using bullet points is recommended.



12
MODEL QUESTION PAPER
FINAL EXAMINATION
SESSION : 2023 - 24
SUBJECT : BIOLOGY
CLASS - XI
Time Allotted : 3 Hours Maximum Marks: 70
General Instructions:
 All questions are compulsory.
 The question paper has five sections and 33 questions. All questions are
compulsory.
 Section–A has 16 questions of 1 mark each.
 Section–B has 5 questions of 2 marks each.
 Section– C has 7 questions of 3 marks each.
 Section– D has 2 case-based questions of 4marks each.
 Section–E has 3 questions of 5 marks each.
 There is no overall choice. However, internal choices have been
provided in some Questions.
 A student must attempt only one of the alternatives in such questions.
 Wherever necessary, neat and properly labelled diagrams should be
drawn.
________________________________________________________________
SECTION – A
1. The chromosomes get aligned at the equator during:
a. prophase b. metaphase
c. anaphase d. telophase
2. Metabolism refers to
a. Release of energy b. Gain of energy
c. Catabolism d. Gain or release of energy
3. Which of the following statements is false about the fungi?
a. They are eukaryotes
1
b. They are heterotrophs
c. They possess a purely cellulosic cell wall
d. None of the above
4. Pteridophytes differ from mosses in
a. Independent gametophyte
b. Dependent gametophyte
c. Flagellate antherozoids
d. Independent and dominant sporophyte
5. Pseudocoelom is found in:
a. cnidaria b. platyhelminthes
c. nematodes d. annelids
6. Ribose sugar belongs to the following group:
a. tetroses b. pentoses
c. hexoses d. heptulose
7. The morphological nature of the edible part of a coconut is
a.Cotyledon b. Perisperm
c. Pericarp d. Endosperm
8. _________ is a product of aerobic respiration
a. Malic acid b. Pyruvate
c. Ethylene d. Lactose
9. Exchange of gases in earthworm occurs by:
a. spiracles b. gills
c. trachaea d. skin
10. White blood cells are also called
a. leucocytes b. erythrocytes
c. thrombocytes d. granulocytes
11. More ADH means:
a. less permeable DCT b. concentrated urine
c. dilute urine d. fall in blood osmotic pressure
12. ____________ tissues synthesize natural cytokinin.
a. Old b. Rapidly dividing
c Storage d. None of the above

2
Question No. 13 to 16 consist of two statements – Assertion (A) and Reason (R).
Answer thesequestions selecting the appropriate option given below:
a) Both A and R are true and R is the correct explanation of A.
b) Both A and R are true and R is not the correct explanation of A.
c) A is true but R is false.
d) A is false but R is true.
13. Assertion: First seven pairs of ribs are called true ribs.
Reason: These ribs are not connected ventrally to the sternum.
14. Assertion: The only product of light reaction, required in dark reaction are NADPH2 and
ATP.
Reason: Dark reaction occurs in night only.
15. Assertion-All motor neurons are efferent neurons.
Reason- the motor neurons carry nerve impulse from spinal cord to brain.
16. Assertion: Tea plants are decapitated for hedge making.
Reason: Removal of stem tip results in the growth of lateral buds.

SECTION – B
17. State the location and function of mesorchium. (2)
18. Briefly explain the law of limiting factors with a suitable example. (2)
19. What is the basic unit of a protein? How are these units linked together? (1+1)
20. Give two significance of mitosis. (2)
21. Throw some light on the dual role of RuBisCo. (2)

SECTION – C
22. Differentiate between:
a. inspiration and expiration
b. aerobic and anaerobic respiration (3)
23. Define hypertension. What is the cause of hypertension? (3)
24. What is tubular secretion. Write down the function of the Henle’s loop. (1+2)
25. Define the following.
a. exocrine gland
b. endocrine gland
c. hormone (3)
26. Write a short note on vascular tissue system. (3)
27. Define CSF. State its location and function. (3)

3
28. Explain alcoholic fermentation. (3)

SECTION – D

29.
a. Name the process shown in the above diagram.
b. Where is vasa recta present?
c. What is uremia?
d. Write the full form of ANF.
OR
d. Name the enzyme secreted by JGA cells (4)
30. The root is usually an underground part of the plant. It is primarily responsible for the
fixation and absorption of water. The root with its branches is known as the root system.
Without the roots, the transportation system in many plants would cease to exist.
Therefore, it is important that you understand the need for roots. However, there are
more than one types of root systems in plants.
(i) What are the roots that grow from any other part of the plant other than the radicle
known as?
(ii) There are massive aerial roots present in a Banyan tree. What are they called?
(iii) Plants growing in swamps have roots that grow vertically upwards likeconical spikes and
have aerating pores. What are such roots called?
(iv) The roots of the parasitic plant Cuscuta show presence of ……. roots.
OK
(iv) A large globular root that tapers sharply at the lower end is called? (1×4=4)
31. a. Diagrammatically represent sliding filament theory.
b. State the two distinct regions of a myosin unit.
OR

4
a. Differentiate between
i) actin and myosin filaments.
ii) A band and I band
32. (i) Define RQ. What is its value for fats?
(ii) Write the significance of citric acid cycle.
OR
What are the main steps of aerobic respiration? Where does it take place? (5)
33. a. Draw a well labelled diagram of interphase stage in animal cell.
b. Which stage comes after interphase?
c. Name the stage when DNA synthesis occurs.
OR
a. All vertebrates are chordates but all chordates are not vertebrates”- Justify the
statement.
b.What type of circulatory system do annelids have?
c.Name the excretory organ of the platyhelminthes.

ANSWER
SECTION – A

1. (b)
2. (d)
3. (c)
4. (d)
5. (b)
6. (b)
7. (d)
8. (b)
9. (d)
10. (a)
11. (b)
12. (b)
13. (b)
14. (c)
15. (c)
16. (a)

5
SECTION – B
17. In males the testis is an elongated organ posteriorly in the pleuroperitoneal cavity lying
ventral to the kidney. It is supported by the mesorchium.
18. Blackman gave the Law of Limiting factors. When several factors affect any biochemical
process, then this law comes into effect. This states that:
If a chemical process is affected by more than one factor, then its rate will be determined
by the factor which is nearest to its minimal value. It is the factor which directly affects
the process if this quantity is changed.
To illustrate the law, suppose light intensity supplied to a leaf is just sufficient to utilize 5
mg of CO2 per hour in photosynthesis. As the CO2 supply is increased, the rate also
increases to 5 mg of CO2 enters the leaf per hour. After that, any further increase in the
supply of CO2 does not have any effect on the rate. Light has now become the limiting
factor and further increase in the rate of photosynthesis will occur only by increasing the
intensity of light.
19. Amino acids.
These units are linked with the help of peptide bonds.
20. It helps in maintaining the same number of chromosomes in daughter cells after division.
It is responsible for growth and development of multicellular organisms. It helps in
repairing of damaged tissues. It helps the cell to maintain proper size. (any two relevant
points)
21. It has a great affinity for CO2 and O2 and the binding is competitive. This shift in roles
occurs when the concentration of either CO2 or O2 increases. It is the relative
concentration of CO2 and O2 that determines which of the two will bind to the enzyme.
RuBisCO carries out more carboxylation in C4 plants because these plants have a
mechanism that increases the concentration of CO2 at the enzyme site. So, RuBisCO
functions as a carboxylase minimizing the oxygenase activity.

SECTION – C

22. a.

6
b.

23. High blood pressure usually develops over time. It can happen because of unhealthy
lifestyle choices, such as not getting enough regular physical activity. Certain health
conditions, such as diabetes and having obesity, can also increase the risk for developing
high blood pressure.
24. The filtrate absorbed in the glomerulus flows through the renal tubule, where nutrients
and water are reabsorbed into capillaries. At the same time, waste ions and hydrogen
ions pass from the capillaries into the renal tubule. This process is called secretion.
The function is to reabsorb the water and the sodium chloride from the filtrate. This
helps to conserve water for the organism, which results in highly concentrated urine.
25. (a) Exocrine glands are the glands which discharge their secretions via ducts onto the
body surface or into the cavities in the body. (b) Endocrine glands are the ductless
glands. These pass their secretions into the surrounding blood for transport to the site of
action.
(c) Hormone is a substance that is synthesised and secreted in very small quantities into
the bloodstream by an endocrine gland or a specialized nerve cell and regulates the
growth or functioning of a specific tissue organ in a distant part of the body e.g. insulin.
26. Vascular tissue is comprised of the xylem and the phloem, the main transport systems of
plants. They typically occur together in vascular bundles in all plant organs, traversing
roots, stems, and leaves.
Xylem is responsible for the transport of water and dissolved ions from the roots
upwards through the plant. Phloem transports metabolites (mainly sugars, amino acids,
and some ions) in solution from “sources” of production, mainly fully expanded leaves,
to “sinks,” including developing roots, leaves, and fruits.
Xylem consists of tracheid, vessels, parenchyma, and fibres.
The phloem is comprised of sieve tubes, companion cells, parenchyma cells, and fibres.
Sieve tubes are separated into sieve tube members, commonly referred to as sieve
elements, by thickened end walls, termed “sieve plates,” pierced by sieve pores.
27. Cerebrospinal fluid (CSF) is a clear, plasma-like fluid (an ultrafiltrate of plasma) that
bathes the central nervous system (CNS). It occupies the central spinal canal, the
ventricular system, and the subarachnoid space.CSF performs vital functions including:
Support; Shock absorber; Homeostasis; Nutrition; Immune function.

7
28. Alcoholic fermentation is a process by which sugar is converted into ethyl alcohol and
CO2 by some bacteria and yeast. First, sugar is converted into pyruvic acid, which is then
converted into ethanol and carbon dioxide by pyruvate decarboxylase and alcohol
dehydrogenase. This process is utilised in the production of alcoholic beverages and
bread.

SECTION – D
29. a. counter current mechanism
b. Vasa recta are the series of long loops of thin-walled blood vessels (efferent arterioles)
that dip down alongside the loop of Henle in the vertebrate kidney. These capillaries are
hairpin-shaped blood vessels and they run parallel to the loops of Henle.
c. Uremia is the disorder of excretory system . It is the malfunctioning of kidneys which
can lead to the accumulation of urea in blood which is highly harmful and may lead to a
kidney failure. In such patients, urea can be removed by a process called hemodialysis
d. Atrial natriuretic factor
OR
d. renin
30. (i) Adventitious roots
(ii) Prop roots
(iii) Pnematophores
(iv)Haustoria/ sucking roots
OR
(iv) Napiform roots

8
SECTION – E

31. a.
b. Myosin consists of an elongated tail region attached to a globular head via a flexible
neck structure.
OR
a.

9
b.
32.i) Respiratory Quotient (RQ) : The ratio of the volume of CO2 evolved to the volume of
O2 consumed in respiration is termed as the respiratory quotient or respiratory ratio.

Its value for fats is less than one.

(ii) Significance of Citric acid cycle


a) It explains the process of breaking of pyruvate into CO2 and water.
It is major pathway of generation of ATP.
b) More energy is released (30 ATP) in this process as compared to glycolysis.
c) Many intermediates compounds are formed. They are used in the synthesis of other
biomolecules like amino-acids, nucleotides, chlorophyll, cytochromes and fats.
OR
Aerobic respiration occurs in main four steps, namely glycolysis, link reaction, Krebs
cycle and terminal oxidation.
Glycolysis: It is partial oxidation of glucose to form two molecules of pyruvate, two
NADH, and two ATP. Glycolysis occurs in the cytoplasm.
Link Reaction: After entering mitochondria, pyruvate undergoes oxidative
decarboxylation to form acetyl CoA and NADH. CO2 is released.
Krebs Cycle: Acetyl CoA reacts with oxaloacetate. The product, citrate, undergoes two
oxidative decarboxylations and two dehydrogenations. Oxaloacetate is regenerated. The
by products are 3 NADH, 1 FADH2, 1 ATP or GTP and 2CO2. Krebs Cycle occurs
inside mitochondria.
Terminal Oxidation: NADH and FADH are oxidized. The energy liberated during
oxidation is used in the synthesis of ATP and ADP and inorganic phosphate. Hydrogen is

10
ultimately oxidized by oxygen to form water. Terminal oxidation takes place over the
inner membrane of mitochondria.

33. a.
b. mitotic phase
c. S- phase
OR
a. In vertebrates notochord is replaced by the vertebral column (backbone), however, the
vertebral column is not present in a protochordate. Therefore, all vertebrates are
chordates but all chordates are not vertebrates.
b. Closed circulatory system
c. Flame cells



11

You might also like